Re: [obm-l] Teorema de Dirichlet

2002-04-18 Por tôpico Alexandre F. Terezan

a1 = 3 (primo)
r = 3 (primo)

No entanto, tal PA só possui um único termo que primo, que é o próprio 3...

-Mensagem Original-
De: Ricardo Miranda [EMAIL PROTECTED]
Para: [EMAIL PROTECTED]
Enviada em: Quinta-feira, 18 de Abril de 2002 20:06 Terezan
Assunto: [obm-l] Teorema de Dirichlet


Alguem pode me mostrar a demonstração do Teorema de Dirichlet, que, se nao
me engano, diz que se numa PA, o primeiro termo e a razao sao primos,
existem infinitos primos dentre os elementos desta PA?


=
[]s
--
Ricardo Miranda
[EMAIL PROTECTED]
http://rm2.hpg.ig.com.br/


___
Yahoo! Empregos
O trabalho dos seus sonhos pode estar aqui. Cadastre-se hoje mesmo no Yahoo!
Empregos e tenha acesso a milhares de vagas abertas!
http://br.empregos.yahoo.com/
=
Instruções para entrar na lista, sair da lista e usar a lista em
http://www.mat.puc-rio.br/~nicolau/olimp/obm-l.html
O administrador desta lista é [EMAIL PROTECTED]
=


=
Instruções para entrar na lista, sair da lista e usar a lista em
http://www.mat.puc-rio.br/~nicolau/olimp/obm-l.html
O administrador desta lista é [EMAIL PROTECTED]
=



Re: Re:[obm-l] como posso resolver ?

2002-03-15 Por tôpico Alexandre F. Terezan

Abaixo utilizarei as incógnitas X, n, m, k, t, a, todos inteiros.

Seja X = 5^n + n^5

Para X ser múltilplo de 13,  5^n + n^5 == 0 (mod 13), ou seja:

5^n == -n^5 (mod 13)(conclusao 1)

5^2 == 25 == (-1) (mod 13)
5^3 == (-5) (mod 13)
5^4 == 1 (mod 13)
5^(4m) == 1^m == 1 (mod 13)(conclusao 2)
5^(4m+1) == 5 (mod 13) (conclusao 3)
5^(4m+2) == 25 == (-1) (mod 13)  (conclusao 4)
5^(4m+3) == (-5) (mod 13) (conclusao 5)

5^12 == 1^3 == 1 (mod 13)
5^13 == 5 (mod 13)
5^(13k) == 5^k (mod 13)
5^(13k+a) == 5^(k+a) (mod 13)(Conclusao 6)



Dividindo em casos:

1) n = 13k ; n == 0 (mod 13)  -- n^5 == 0 (mod 13) ; 5^(13k) == 5^k (mod
13)

Da conclusao 1, vem:  5^k == 0 (mod 13)  (IMPOSSÍVEL)

2) n = 13k + 1; n == 1 (mod 13) -- n^5 == 1^5 == 1 (mod 13) ; 5^(13k+1) ==
5^(k+1) (mod 13)

Da conclusao 1, vem:  5^(k+1) == (-1) (mod 13)

Da conclusao 4, vem que (k+1) = (4m + 2), ou seja, k = 4m + 1

Mas n = 13k+1, logo n = 13(4m+1) + 1  = 52 m + 14 (1a SOLUCAO)

3) n = 13k + 2; n == 2 (mod 13) -- n^5 == 2^5 == 6 (mod 13) ; 5^(13k+2) ==
5^(k+2) (mod 13)

Da conclusao 1, vem:  5^(k+2) == (-6) (mod 13)   (IMPOSSÍVEL)

4) n = 13k + 3; n == 3 (mod 13) -- n^5 == 3^5 == 9 (mod 13) ; 5^(13k+3) ==
5^(k+3) (mod 13)

Da conclusao 1, vem:  5^(k+3) == (-9) (mod 13)   (IMPOSSÍVEL)

5) n = 13k + 4; n == 4 (mod 13) -- n^5 == 4^5 == 10 (mod 13) ; 5^(13k+4) ==
5^(k+4) (mod 13)

Da conclusao 1, vem:  5^(k+4) == (-10) == 3 (mod 13)   (IMPOSSÍVEL)

6) n = 13k + 5; n == 5 (mod 13) -- n^5 == 5^5 == 5 (mod 13) ; 5^(13k+5) ==
5^(k+5) (mod 13)

Da conclusao 1, vem:  5^(k+5) == (-5) (mod 13)

Da conclusao 5, vem que (k+5) = (4m + 3), ou seja, k = 4m - 2

Mas n = 13k+5, logo n = 13(4m-2) + 5  = 52 m - 21 (2a SOLUCAO)

7) n = 13k + 6; n == 6 (mod 13) -- n^5 ==  6^5 == 2 (mod 13) ; 5^(13k+6) ==
5^(k+6) (mod 13)

Da conclusao 1, vem:  5^(k+6) == (-2) (mod 13)   (IMPOSSÍVEL)

8) n = 13k + 7; n == 7 (mod 13) -- n^5 == 7^5 == 11 (mod 13) ; 5^(13k+7) ==
5^(k+7) (mod 13)

Da conclusao 1, vem:  5^(k+7) == (-11) == 2 (mod 13)   (IMPOSSÍVEL)

9) n = 13k + 8; n == (-5) (mod 13) -- n^5 == (-5)^5 == (-5) (mod 13) ;
5^(13k+8) == 5^(k+8) (mod 13)

Da conclusao 1, vem:  5^(k+8) == 5 (mod 13)

Da conclusao 3, vem que (k+8) = (4m + 1), ou seja, k = 4m - 7

Mas n = 13k+8, logo n = 13(4m-7) + 8  = 52 m - 83 (3a SOLUCAO)

10) n = 13k + 9; n == (-4) (mod 13) -- n^5 == (-4)^5 == (-10) (mod 13) ;
5^(13k+9) == 5^(k+9) (mod 13)

Da conclusao 1, vem:  5^(k+9) == 10 (mod 13)   (IMPOSSÍVEL)

11) n = 13k + 10; n == (-3) (mod 13) -- n^5 == (-3)^5 == (-9) (mod 13) ;
5^(13k+10) == 5^(k+10) (mod 13)

Da conclusao 1, vem:  5^(k+10) == 9 (mod 13)   (IMPOSSÍVEL)

12) n = 13k + 11; n == (-2) (mod 13) -- n^5 == (-2)^5 == (-6) (mod 13) ;
5^(13k+11) == 5^(k+11) (mod 13)

Da conclusao 1, vem:  5^(k+11) == 6 (mod 13)   (IMPOSSÍVEL)

13) n = 13k + 12; n == (-1) (mod 13) -- n^5 == (-1)^5 == (-1) (mod 13) ;
5^(13k+12) == 5^(k+12) (mod 13)

Da conclusao 1, vem:  5^(k+12) == 1 (mod 13)

Da conclusao 2, vem que (k+12) = 4m , ou seja, k = 4m - 12

Mas n = 13k+12, logo n = 13(4m-12) + 12  = 52 m - 144 (4a SOLUCAO)


Assim, vejamos a congruência das solucoes de n módulo 52...

1a SOLUCAO:   n == 14 (mod 52)
2a SOLUCAO:   n == (-21) == 31 (mod 52)
3a SOLUCAO:   n == (-83) == 21 (mod 52)
4a SOLUCAO:   n == (-144) == 12 (mod 52)

Assim, para todo t inteiro, as solucoes sao:

(n = 52t + 12;  n = 52t + 14; n = 52t + 21; n = 52t + 31)

Logo, as primeiras solucoes entre inteiros positivos sao:

n = 12; 14; 21; 31; 64; 66; 73; 83; etc

Espero ter ajudado...
[ ]'s
Alexandre Terezan

PS: Houve algum erro com seu script...








-Mensagem Original-
De: Helder Suzuki [EMAIL PROTECTED]
Para: [EMAIL PROTECTED]
Enviada em: Quinta-feira, 14 de Março de 2002 21:22 Terezan
Assunto: Re:[obm-l] como posso resolver ?


obrigado por responder o meu e-mail.
eu fiz um pequeno script e encontrei vários n's.
mas não sei como calcular esse ciclo..
lá vai alguns:
n  | 5^n + n^5
12 | 244389457
14 | 6104053449
21 | 476837162287226
25 | 298023223886718750
54 | 5,5511151231257827021181583405e+37

 --- dirichlet [EMAIL PROTECTED] escreveu:  Sugestao:use
congruencias.Tente na porrada achar o
 primeiro n.Depois calcule o ciclo desta funçao(5^n+n^5)
 modulo 13(nao se assuste se voce demorar um pouco).E
 pronto!!


 ORIGINAL MESSAGE

  Eu ficaria muito feliz se alguem puder me dar uma luz:
 
  Dê todos os n possíveis para 5^n + n^5 ser multiplo de
 13.
 
  Hélder
 
 
 _
 __
  Yahoo! Empregos
  O trabalho dos seus sonhos pode estar aqui. Cadastre-
 se hoje mesmo no Yahoo! Empregos e tenha acesso a
 milhares de vagas abertas!
  http://br.empregos.yahoo.com/
 
 =
 
  Instruções para entrar na lista, sair da lista e usar
 a lista em
  

Re: [obm-l] Probabilidade da probabilidade

2002-03-11 Por tôpico Alexandre F. Terezan

Se as amostras sao eventos independentes, porque a a probabilidade da
segunda amostra ser branca nao é (1-p)???

-Mensagem Original-
De: [EMAIL PROTECTED]
Para: [EMAIL PROTECTED]
Enviada em: Segunda-feira, 11 de Março de 2002 22:53 Terezan
Assunto: [obm-l] Probabilidade da probabilidade


Eu pensei num exercicio e achei uma resposta legal, descuti com o humberto
naves e ainda ganhei um sanduiche apostando, porque ele nao acreditou na
resposta, mas depois aceitou. Vou tentar formalizar:
Um evento pode ser preto ou branco, sendo uma probabilidade p de ser preto e
1-p de ser branco, mas nao se sabe p. so se sabe que para todo 0=ab=1, a
probabilidade de p estar no intervalo [a,b] é b-a..
Se a primeira amostra do evento foi branca, qual a probabilidade da segunda
amostra ser branca tambem. tentem ai..
Desculpa qualquer erro no anuciado e se for um execicio ja batido...
Carlos

obs.Pode-se generalizar para n primeiro eventos, sendo k pretos e n-k
brancos, qual a prob. do proximo ser branco.
obs2.ha outra forma de ver o anuciado, mas seria como uma primeira etapa da
resposta, ai facilitaria, entao depois falo se necessario..
=
Instruções para entrar na lista, sair da lista e usar a lista em
http://www.mat.puc-rio.br/~nicolau/olimp/obm-l.html
O administrador desta lista é [EMAIL PROTECTED]
=


=
Instruções para entrar na lista, sair da lista e usar a lista em
http://www.mat.puc-rio.br/~nicolau/olimp/obm-l.html
O administrador desta lista é [EMAIL PROTECTED]
=



[obm-l] Re: [obm-l] funções piso(x) e teto(x)

2002-03-01 Por tôpico Alexandre F. Terezan

O exercício i possui infinitos contra-exemplos.

n = 7 --  lado esquerdo = 2; lado direito = 3
n = 16 -- lado esquerdo = 6; lado direito = 7
etc etc

Na verdade, para todo n = 9k + 7 (k inteiro nao-negativo), a afirmacao é
falsa.
Isto é fácil de demonstrarmos...

Para n = 9k + 7,  piso(2n/3) = piso((18k+14)/3) = piso (6k + 4 + (2/3)) = 6k
+ 4

piso(2*piso(2*n/3)/3) = piso(2*(6k+4)/3) =piso((12k+8)/3) =piso(4k + 2 +
(2/3))= 4k+2

Mas para n = 9k + 7, piso(4n/9) = piso((36k+28)/9) = piso(4k + 3 + (1/9)) =
4k + 3

Como (4k + 2) e (4k + 3) sao naturais diferentes, entao conclui-se que, para
todo n=9k +7,

piso(2*piso(2*n/3)/3) É DIFERENTE DE piso(4*n/9).

OBS.: Utilizando raciocínio semelhante, demonstramos que a afirmacao acima
VALE para todo n DIFERENTE DE 9k + 7...

[ ]'s
Alexandre Terezan.

-Mensagem Original-
De: Antonio Jose Gonzales Alves [EMAIL PROTECTED]
Para: [EMAIL PROTECTED]
Enviada em: Sexta-feira, 1 de Março de 2002 09:52 Terezan
Assunto: [obm-l] funções piso(x) e teto(x)



Bom dia pessoal, estou com uma lista de exercícios aqui da faculdade e não
estou conseguindo ter nenhuma idéia para provar ou dar contra-exemplos dos
seguintes exercícios:

i)   piso(2*piso(2*n/3)/3) = piso(4*n/9) , n é inteiro positivo

ii)  piso(piso(n/a)/b) = piso(n/(a*b)) , n,a,b são inteiros positivos


obs.: o que eu quero dizer com piso(x) é o único inteiro que satisfaz

piso(x) = x  piso(x) + 1


Se alguém puder me ajudar eu ficaria muito grato,
Um grande abraço a todos,



|=-=-=-=-=-=-=-=-=-=-=-=-=-=-=-|
|--- \\|// |
|--- (o o) |
|--- oOOo~(_)~oOOo |
|--|
|- Toninho :\ |
|[EMAIL PROTECTED]|
 |--|


=
Instruções para entrar na lista, sair da lista e usar a lista em
http://www.mat.puc-rio.br/~nicolau/olimp/obm-l.html
O administrador desta lista é [EMAIL PROTECTED]
=


=
Instruções para entrar na lista, sair da lista e usar a lista em
http://www.mat.puc-rio.br/~nicolau/olimp/obm-l.html
O administrador desta lista é [EMAIL PROTECTED]
=



Re: [obm-l] Divisibilidade

2002-02-28 Por tôpico Alexandre F. Terezan

== significa côngruo

2^6 == 64 == 3  (mod 61)
2^24 == 3^4 == 20 (mod 61)
2^48 == 20^2 == 34 (mod 61)
2^48 - 1 == 33 (mod 61)

Logo, 2^48 - 1 nao é divisível por 61.

2^6 == 64 == (-3)  (mod 67)
2^24 == (-3)^4 == 14(mod 67)
2^48 == 14^2 == 62 (mod 67)
2^48 - 1 == 61 (mod 67)

Logo, 2^48 - 1 nao é divisível por 67.

2^6 == 64 == (-5)  (mod 69)
2^18 == (-5)^3 == (-125) == 13 (mod 69)
2^24 == (-5) * 13 == (-65) == 4 (mod 69)
2^48 == 4^2 == 16 (mod 69)
2^48 - 1 == 15 (mod 69)

Logo, 2^48 - 1 nao é divisível por 69.

[ ]'s
Alexandre Terezan.


-Mensagem Original-
De: Angelo Barone Netto [EMAIL PROTECTED]
Para: [EMAIL PROTECTED]
Enviada em: Quinta-feira, 28 de Fevereiro de 2002 19:51 Terezan
Assunto: Re: [obm-l] Divisibilidade




Angelo Barone{\ --\ }Netto   Universidade de Sao Paulo
Departamento de Matematica Aplicada  Instituto de Matematica e Estatistica
Rua do Matao, 1010   Butanta - Cidade Universitaria
Caixa Postal 66 281  phone +55-11-3091-6162/6224/6136
05311-970 - Sao Paulo - SP   fax +55-11-3091-6131
Agencia Cidade de Sao Paulo
.

2^48 - 1 = 63 * 65 * (2^12 + 1)(2^24 + 1).
2^12 + 1=3^2*5*7*13
2^24 + 1=3^2*5*7*13*17*241
63=3^2*7
65=5*13
2^48 - 1=3^6*5^3*7^3*13^3*17*241 tem
7*4*4*4*2=896 divisores:
  1   3  9  27  81 ...
  5  15 45 135 ...
 25  75 ...
125  ...
  7  21 35  63 105 ...
 49 147 ...
343 ...
 13  39 65 91 ...
169 ...
...
 17  51 85 ...
241 ...
(onde ... indica divisores estritamente maiores do que 70)

 Logo, 63 e 65 sao os unicos números procurados.
Deve haver modo mais inteligente de mostrar qur 61, 67 e 69 nao
dividem 2^48 - 1.

=
Instruções para entrar na lista, sair da lista e usar a lista em
http://www.mat.puc-rio.br/~nicolau/olimp/obm-l.html
O administrador desta lista é [EMAIL PROTECTED]
=


=
Instruções para entrar na lista, sair da lista e usar a lista em
http://www.mat.puc-rio.br/~nicolau/olimp/obm-l.html
O administrador desta lista é [EMAIL PROTECTED]
=



Re: [obm-l] Divisibilidade

2002-02-27 Por tôpico Alexandre F. Terezan



2^48 - 1 = (2^24 - 1)(2^24 + 1) = (2^12 - 1)(2^12 + 1)(2^24 + 
1) = (2^6 - 1)(2^6 + 1)(2^12 + 1)(2^24 + 1) = 63 * 65 *(2^12 + 1)(2^24 + 
1).

Logo, 63 e 65 sao os números procurados.

  -Mensagem Original- 
  De: [EMAIL PROTECTED] 
  Para: [EMAIL PROTECTED] 
  Enviada em: Quarta-feira, 27 de Fevereiro 
  de 2002 19:56 Terezan
  Assunto: [obm-l] Divisibilidade
  Bom dia, 
  gostaria da ajuda de vocês nesse: 
  Há dois números entre 60 e 70 que 
  dividem 2^48 - 1 (dois elevado a quarenta e oito, menos um). Quais são eles? 
  Obrigado, 
  Raul 
  PS : Ainda gostaria muito que alguém tentasse mostrar pelo menos o caminho 
  de demonstração do limite da minha mensagem "Quando existe o limite?". 
  


Re: [obm-l] Muito interressante

2002-02-25 Por tôpico Alexandre F. Terezan

SIM, É POSSÍVEL...

Ou seja, podemos escrever qualquer número de (1 - 3^n)/2   a   (3^n - 1)/2
com no máximo n algarismos (-1, 0 ou 1) na base 3.

Demonstracao:

1) Se vale de 0 a (3^n - 1)/2, vale de (1 - 3^n)/2 a 0:  (conclusao I)

Para verificar isto, basta trocarmos (-1) por (1) e (1) por (-1), mantendo o
(0).

2) Vale para 0 a (3^n - 1)/2?  -- Princípio da Inducao

 2.1) Vale para n = 1 --  podemos escrever 0 como 0 (base 3) e 1 como
1 (base 3)

 2.2) Se vale para n, vale para n+1:

   Ora, com n algarismos podemos escrever todos os números entre 0 e (3^n -
1)/2.
Assim, adicionando um algarismo 1 na posicao n+1 (o que nos dá 3^n na base
10), e sabendo que podemos formar qualquer número de (1 - 3^n)/2  a  0
(vide conclusao I), fica claro que formamos assim qualquer número de:

3^n + (1-3^n)/2  a  3^n, ou seja,  de  (3^n + 1)/2  a  3^n.

Da mesma maneira, podemos adicionar a 3^n os números formados de 0 a
(3^n -1)/2, obtendo todos os números de:

3^n  a  3^n + (3^n - 1)/2, ou seja,   de  3^n  a  (3^(n+1) - 1)/2

Entao, podemos formar todos os números de 0 a (3^n - 1)/2  e de (3^n + 1)/2
a  (3^(n+1) - 1)/2.

Como (3^n - 1)/2 e  (3^n + 1)/2  sao naturais consecutivos, podemos formar
qualquer número de:

  0  a  (3^(n+1) - 1)/2  com (n+1) algarismos, o que conclui a prova por
inducao.

Espero ter ajudado...
[ ]'s

Alexandre Terezan





-Mensagem Original-
De: Jose Jayme Moraes Junior [EMAIL PROTECTED]
Para: [EMAIL PROTECTED]
Enviada em: Segunda-feira, 25 de Fevereiro de 2002 10:34 Terezan
Assunto: RE: [obm-l] Muito interressante



Isto também funciona para inteiros de -121 a +121 (1,3,9,27 e 81)
utilizando na base 3 com os algarismos -1,0,+1 ?? Pelos exemplos abaixo,
sim.
Exemplos:
41 = 81 - 27 - 9 - 3 - 1
42 = 81 - 27 - 9 - 3
45 = 81 - 27 - 9
50 = 81 - 27 - 3 - 1
58 = 81 - 27 + 3 + 1
60 = 81 - 27 + 9 - 3
75 = 81 - 9  + 3

É possível estender para 3^n ? 1, 3, 9, 27, 81, ., 3^n


-Original Message-
From: [EMAIL PROTECTED]
[mailto:[EMAIL PROTECTED]] On Behalf Of Nicolau C.
Saldanha
Sent: domingo, 24 de fevereiro de 2002 09:49
To: [EMAIL PROTECTED]
Subject: Re: [obm-l] Muito interressante


On Fri, Feb 22, 2002 at 02:29:11PM -0500, [EMAIL PROTECTED] wrote:
Oi pessoal,
uma professora me apresentou um problema interessante criado
 por ela e
 cuja solução é ainda mais interessante. Queria saber se há alguma
regra que
 explica essa solução tão curiosa.
Problema : Um feirante possuía uma balança de pratos e quarenta
pesos
 numerados de um até 40 que indicava a massa que ele vendia (de um a
quarenta
 quilos). O peso de 40 quilos caiu e quebrou em 4 partes. Um matemático
que
 queria montar uma barraca ,mas não tinha peso algum, observou (pesou)
as
 partes quebradas e pediu-as. Com elas o matemático conseguia pesar com
a
 mesma precisão massas de 1 a 40 quilos. Quais as massas das partes?
Solução : 1, 3, 9 e 27.

O matemático observa que todo inteiro de -40 a 40 pode ser escrito na
base 3 com os algarismos -,0,+ (-1, 0 e 1) usando no máximo 4
algarismos. Por exemplo:

-5 = 0-++ =- 9 + 3 + 1
13 = 0+++ =  9 + 3 + 1
20 = +-+- = 27 - 9 + 3 - 1

Não sei se é tão fácil verificar se esta (1,3,9,27) é a única solução.
[]s, N.

=
Instruções para entrar na lista, sair da lista e usar a lista em
http://www.mat.puc-rio.br/~nicolau/olimp/obm-l.html
O administrador desta lista é [EMAIL PROTECTED]

=

=
Instruções para entrar na lista, sair da lista e usar a lista em
http://www.mat.puc-rio.br/~nicolau/olimp/obm-l.html
O administrador desta lista é [EMAIL PROTECTED]
=


=
Instruções para entrar na lista, sair da lista e usar a lista em
http://www.mat.puc-rio.br/~nicolau/olimp/obm-l.html
O administrador desta lista é [EMAIL PROTECTED]
=



Re: [obm-l] russos

2002-01-24 Por tôpico Alexandre F. Terezan

Faltam ainda diversos casos a considerar, mas é por aí mesmo...

Eu consegui resolver o problema, se ninguem resolver eu mando a resposta...

-Mensagem Original-
De: Iolanda Brazão [EMAIL PROTECTED]
Para: [EMAIL PROTECTED]
Enviada em: Quinta-feira, 24 de Janeiro de 2002 14:26 Terezan
Assunto: Re: [obm-l] russos


Oi gente.

No problema 1, notar que quando se passa de um natural N para o natural N+1
a soma dos algarismos - de N - passa de A para A+1, desde que N nao tenha 9
para algarismo das unidades. Se 9 for o algarismo das unidades de N entao a
soma dos algarismos de N+1 sera A-9K+1, para algum K inteiro. Neste ultimo
caso, a soma dos algarismo de N+1 deve deixar resto 1 quando divisivel por
11 e como sao 39 numeros, isso devera acontecer, ao menos, 3 vezes ...
Ajudou ?


From: gabriel guedes [EMAIL PROTECTED]
Reply-To: [EMAIL PROTECTED]
To: [EMAIL PROTECTED]
Subject: [obm-l] russos
Date: Wed, 23 Jan 2002 16:28:27 -0200

Ola amigos da lista,
estava resolvendo alguns problemas russos ( aqueles que o Paulo
traduziu), mas estou com dificuldades nesse dois:

1)Prove que em qualquer sequencia de 39 numeros naturais consecutivos
existe ao menos  um numero cuja a soma dos algarismos e divisivel por
11.
2)Dados quaisquer numeros naturais  m ,n e k' . prove que  nós
sempre  podemos  encontrar dois numeros r e s, primos entre si , tal
que r*m + s*n é um multiplo de k.

Agradeo desde  a QUALQUER colaborao,
Gabriel.





_
Associe-se ao maior serviço de e-mail do mundo através do MSN Hotmail.
http://www.hotmail.com/br

=
Instruções para entrar na lista, sair da lista e usar a lista em
http://www.mat.puc-rio.br/~nicolau/olimp/obm-l.html
O administrador desta lista é [EMAIL PROTECTED]
=


=
Instruções para entrar na lista, sair da lista e usar a lista em
http://www.mat.puc-rio.br/~nicolau/olimp/obm-l.html
O administrador desta lista é [EMAIL PROTECTED]
=



[obm-l] Re: [obm-l] Re: Orientação para resolução

2002-01-21 Por tôpico Alexandre F. Terezan

2^4 == 16 (mod 100)
2^12 == 96 == -4 (mod 100)
2^60 == (-4)^5 == 24 (mod 100)
2^72 == (-4)^6 == 96 == -4 (mod 100)
2^432 == (-4)^6 == 96 == -4 (mod 100)
2^864 == (-4)^2 == 16 (mod 100)

2^936 == 2^864 * 2^72 == 16 * (-4) == 64 == -36 (mod 100)
2^996 == 2^936 * 2^60 == (-36) * 24 == -64 == 36 (mod 100)

2^1000 == 2^996 * 2^4 == 36 * 16 == 76 (mod 100)

-Mensagem Original-
De: [EMAIL PROTECTED]
Para: [EMAIL PROTECTED]
Enviada em: Segunda-feira, 21 de Janeiro de 2002 12:55 Terezan
Assunto: [obm-l] Re: Orientação para resolução


Quais são os últimos dois algarismos de 2^1000 ??


Não sei resolver esse tipo de questão, mas como encontrei a resposta certa
resolvi mandar a mensagem !!

Será que alguém poderia postar uma maneira mais fácil de obter essa resposta
??


obs : as igualdades são todas mod 100

2^10 = 24
(2^10)^100 = 24^100 = (2^3*3)^100 =

(2^10)^30*3^100 = 24^30*3^100 = (2^3*3)^30*3^100 =

(2^10)^9*3^130 = 24^9*3^130 = (2^3*3)^9*3^130 =

2^27*3^139 = 2^30/8*3^139 = 24^3/8*3^139 = 12^3*3^139 =

4^3*3^142 = 64*3^142

analisando as potencias de 3 mod 100:

3^1=3
3^2=9
3^3=27
3^4=81
3^5=43
3^6=29
3^7=87
3^8=61
3^9=83
3^10=49



3^142*64=49^14*64*9


analisando as potencias de 49 mod 100:

49^1=49
49^2=01
49^3=49
49^4=01
..
49^14=01 mod 100

assim, temos que : 49^14*64*9 = 64*9 = 76 mod 100

Portanto os últimos dois algarismos de 2^1000 é 76
conferindo :

2^1000 =
1071508607186267320948425049060001810561404811705533607443750388370351051124
9361224931983788156958581275946729175531468251871452856923140435984577574698
5748039345677748242309854210746050623711418779541821530464749835819412673987
67559165543946077062914571196477686542167660429831652624386837205668069376



Mathematicus nascitur, non fit
Matemáticos não são feitos, eles nascem


--
Use o melhor sistema de busca da Internet
Radar UOL - http://www.radaruol.com.br



=
Instruções para entrar na lista, sair da lista e usar a lista em
http://www.mat.puc-rio.br/~nicolau/olimp/obm-l.html
O administrador desta lista é [EMAIL PROTECTED]
=


=
Instruções para entrar na lista, sair da lista e usar a lista em
http://www.mat.puc-rio.br/~nicolau/olimp/obm-l.html
O administrador desta lista é [EMAIL PROTECTED]
=



Re: Somatório dos primeiros impares

2002-01-10 Por tôpico Alexandre F. Terezan

O n-ésimo ímpar pode ser representado por 2n-1

Assim a soma dos termos desta PA de razao 2 é:

(a1+an)*(n/2) = (1+2n-1)*(n/2) = n^2

-Mensagem Original-
De: Ricardo Miranda [EMAIL PROTECTED]
Para: [EMAIL PROTECTED]
Enviada em: Quinta-feira, 10 de Janeiro de 2002 22:59 Terezan
Assunto: Somatório dos primeiros impares


Amigos,

Li sobre uma regra de Pitágoras para se calcular a soma dos n primeiros
números impares, por n^2.
Ex: A soma dos 9 primeiros números impares é 9^2 = 81.
Achei interessante a simplicidade da fórmula.. Tentei chegar a ela usando
a formula da soma dos n numeros de uma PA, mas nao consegui, alguem pode me
ajudar?





Re: Urgente Vestibular UFRGS-2002

2002-01-08 Por tôpico Alexandre F. Terezan

a=b=4 nao é a única solucao para a equacao, entendeu?

-Mensagem Original-
De: Thomas de Rossi [EMAIL PROTECTED]
Para: Obm-l [EMAIL PROTECTED]
Enviada em: Terça-feira, 8 de Janeiro de 2002 16:38 Terezan
Assunto: Urgente Vestibular UFRGS-2002


Pessoal,

uma questão do vestibular da UFRGS de 2002 que não estou acreditando estar
certa,
mas conforme o gabarito divulgado está. Se realmente for correta a resposta
dada, gostaria de saber
onde estou cego para não enxergá-la.

04) Considere as proposições abaixo:

( I )125% de 1/5 é igual a 1/4.
( II )Se 1/a + 1/b = 1/2, então a=b=4.
( III )20 m/s correspondem a 72 km/h.

Analisando as proposições conclui-se que:

Resposta dada como correta: (C) apenas I e III são verdadeiras.

NOTA: as proposições I e III estão ok, mas vejam a II? Pra mim está ok
também!!!

Em tempo: Gostaria de compreender melhor o assunto lugar geométrico, não
faço idéia de como se
reseolve isso. Alguém poderia sugerir algum livro?

Valeu, Thomas.





Re: ajuda

2001-12-11 Por tôpico Alexandre F. Terezan



No embalo do que o JP disse, de que só é "bom" usar o que 
demonstramos, e como eu useia desigualdade de Bernoulli na minha solucao, 
a demonstracao abaixo está correta?

(1+x)^n = 1 + nx, para x real maior que -1, 
diferente de zero, e n natural maior que 1.

Para n = 2 -- (1+x)^2 = 1 + 2x + x^2  1 + 
2x (VERDADEIRO)

Inducao: Se vale para n, entao (1+x)^n = 1 + 
nx.

Mas (1+x)^(n+1) = (1+x)^n * (1+x)  (1+nx)(1+x) = 1 + 
(n+1)x + nx^2 1 + (n+1)x

Ou seja, se vale para n natural maior que 1, vale para (n+1) 
também

Como vale para n = 2, entao vale para todo n natural maior que 
1. c.q.d.

  -Mensagem Original- 
  De: Augusto 
  César Morgado 
  Para: [EMAIL PROTECTED] 
  Enviada em: Terça-feira, 11 de Dezembro 
  de 2001 11:32 Terezan
  Assunto: Re: ajuda
  Não há dúvida de que foi linda. Mas, supondo o "sabemos que", 
  bastaria fazer n=1. Alexandre F. Terezan wrote:
  00c301c181e8$703c99a0$[EMAIL PROTECTED] 
type="cite">


Vou tentar uma sem usar cálculo.

Desigualdade de Bernoulli: (1 + a)^n = 1 + 
an, a  -1 e n natural.

Sabemos que e^x  (1 + x/n)^n, para todo n

Seja a = x/n

e^x  (1 + x/n)^n -- e^x  (1 + 
a)^n -- e^x  1 + an -- e^x  1 + 
x

-Mensagem Original- 

  De:[EMAIL PROTECTED] 
  
  Para: 
  [EMAIL PROTECTED] 
  Enviada 
  em: Segunda-feira, 10 de Dezembro de 2001 00:12 Terezan
  Assunto: 
  ajuda
  Como se demonstra a 
  desigualdade e ^ x maior ou igual a 1 + x 
  ?


DÚVIDA

2001-12-06 Por tôpico Alexandre F. Terezan



Alguém poderia me ajudar nessa?

1) Prove que:

k ~=((k^(1/a) + (b-1)) / b)^(ab), onde:

k  1, b  1 e a sendo um 
número suficientemente grande (tendendo ao infinito).


Re: Como simplificar?

2001-12-05 Por tôpico Alexandre F. Terezan



2*3 + 3*5 + 4*7 + 5*9 + 6*11 + ... + (n+1)*(2n+1) =
2*4 - 2 + 3*6 - 3+ 4*8 - 4+ 5*10 - 5+ 6*12 - 6+ ... 
+ (n+1)*(2n+2) - (n+1)=

2*4+ 3*6 + 4*8+ 5*10 + 6*12+ ... + (n+1)*(2n+2)- (2 
+ 3 + 4 + 5 + 6 + ... + (n+1)) =

2*(2^2 + 3^2 + 4^2 + 5^2 + 6^2 + ... + (n+1)^2) - (2 + 3 + 4 + 5 + 6 + ... 
+ (n+1)) =

2*(1^2 + 2^2 + 3^2 + 4^2 + 5^2 + 6^2 + ... + (n+1)^2) - 2 -(2 + 3 + 4 
+ 5 + 6 + ... + (n+1)) =

(2*(n+1)(n+2)(2n+3))/6 - 2 -((n+3)n)/2 =

(n+1)(n+2)(2n+3)/3 - 2 - ((n+3)n)/2 =

(2*(n+1)(n+2)(2n+3) - 12 - 3*((n+3)n)) / 6 =

(4n^3 + 18n^2 + 26n + 12 - 12 - 3n^2 - 9n) / 6 =

(4n^3 + 15n^2 + 17n) / 6 =

(n/6) * (4n^2 + 15n + 
17)



  -Mensagem Original- 
  
  De: Davidson 
  Estanislau 
  Para: obm 
  Enviada em: Quarta-feira, 5 de Dezembro 
  de 2001 10:44 Terezan
  Assunto: Como simplificar?
  
  
   Caros amigos, como 
  faço para simplificar a expressão abaixo?
  
   2*3 
  + 3*5 + 4*7 + 5*9 + 6*11 + ... + (n+1)*(2n+1)
  
   Davidson 
  Estanislau


Re: potencias

2001-12-04 Por tôpico Alexandre F. Terezan



Leia sobre a Representação Binária dos números 
naturais.

I) Todo número pode ser escrito na base 2, utilizando 
algarismos 0 e 1 apenas.

II) Como usar 1 em determinada posicao significa somar a 
potência de 2 correspondente e 0 significa omitir tal potência:

III) "todo numero Natural pode ser escrito como soma de 
potencias de base 2"

Ex: 5 (base 10) = 101 (base 2) = 2^2 + 2^0
 9 (base 10) = 1001 (base 
2) = 2^3 + 2^0
 14 (base 10) = 1110 (base 2) = 
2^3 + 2^2 + 2^1
 43(base 10) =101011 
(base 2) = 2^5 + 2^3 + 2^1 + 2^0 

  -Mensagem 
  Original- 
  De: gabriel 
  guedes 
  Para: [EMAIL PROTECTED] 
  Enviada em: Terça-feira, 4 de Dezembro de 
  2001 13:27 Terezan
  Assunto: potencias
  
  Ola amigos da lista ,
  
  me fizeram a seguinte "todo numero Natural pode ser 
  escrito como soma de potencias de base 2", eu não sei 
  responder.Gostaria da ajuda de todos , se alguem ja 
  viu algum trabalho relacionado a issoqualquer coisa 
mesmo


Re: ajuda (ERRATA)

2001-12-04 Por tôpico Alexandre F. Terezan

Resposta do problema 3:

(XYZ) representa o angulo entre os segmentos de reta XY e YZ.

Seja AQB um triângulo com AQ = 6, BQ = 10, AB = k, com Q exterior a ABC.

Ora, AQB é semelhante a APC (na verdade, sao até congruentes), donde:

  (BAQ) = (CAP) (I)

Como o triângulo ABC é equilátero:

 (CAB) = 60 graus = (CAP) + (PAB)(II)

De I e II vem:

60 graus = (CAP) + (PAB) = (BAQ) + (PAB) = (PAQ)   (III)

Do triângulo APQ vem;

(AQP) + (QPA) + (PAQ) = 180 graus (IV)

Mas AQ = AP, logo:

(AQP) = (QPA) = x(V)

De III, IV e V, vem:

x + x + 60 = 180  --  2x = 120  -- x = 60 graus, donde:

O triângulo AQP é equilátero de lado 6  -- AQ = AP = PQ = 6

Como PQ = 6, BP = 8 e BQ = 10, aplicamos a lei dos cossenos em (BPQ):

10^2 = 6^2 + 8^2 - 2*6*8*cos((BPQ))

2*6*8*cos((BPQ)) = 0  --  cos((BPQ)) = 0 (VI)

Como (BPQ) está entre 0 e 180 graus (exclusos), de VI podemos concluir que:

(BPQ) = 90 graus

(BPA) = (BPQ) + (APQ) = 90 + 60 = 150 graus

Aplicando a lei dos cossenos em (BPA), vem:

k^2 = 6^2 + 8^2 - 2*6*8*cos(150)

k^2 = 36 + 64 - 96 * (-sqrt3)/2 = 100 + 48sqrt3

k = sqrt(100 + 48sqrt3)  ~=  13,53286514

Espero ter ajudado,

[ ]'s

Alexandre Terezan








-Mensagem Original-
De: Vinicius José Fortuna [EMAIL PROTECTED]
Para: [EMAIL PROTECTED]
Enviada em: Terça-feira, 4 de Dezembro de 2001 13:02 Terezan
Assunto: Re: ajuda (ERRATA)


On Mon, 3 Dec 2001, Vinicius José Fortuna wrote:

Ops!, Cometi alguns equívocos:

  2) Qual o 496o termo da sequencia 1,2,2,3,3,3,4,4,4,4,5,5,5,5,5, ...?

  Nessa sequência, o último número n aparece na posição n(n+1)/2, que é o
 somatório de 1 a n. Então para descobrirmos o 496o. termo, devemos
 resolver a inequação:
   x(x+1)/2 = 496, x0

 Isso dá
   x =  31,5

 Portanto o 496o. termo da sequência é 32

Errei as contas. É x=31. Então o 496o. termo é 31.


  3) No interior do triângulo ABC, equilátero, existe um ponto P tal que
  AP = 6, BP = 8 e CP= 10. Determine o perímetro do triângulo ABC.

 Se eu não me engano, existe um teorema que diz que a soma das distâncias
 de qquer ponto interior de um triângulo equilátero é constante e igual a
 2 x tamamnho do lado. Alguém pode confirmar isso pra mim?

Na verdade o teorema que eu estava pensando diz que a soma distâncias de
qquer ponto interior a todos os lados é constante e igual à altura.
Mas aí não dá para resolver de forma tão direta. :-(

Até mais

Vinciius





Re: Traducao dos Problemas Russos

2001-12-04 Por tôpico Alexandre F. Terezan

Olá Paulo e demais integrantes da lista.
Eu nao sei se alguém já respondeu ao problema antes, mas lá vai uma
tentativa.

Gostaria que comentassem, minha solucao é tao elementar que acho q está
errada, hehehe

Imaginemos separadamente cada um dos 5 polígonos delimitados.

2 deles sao verdadeiros retangulos, cada um com 4 arestas. Mas há 3 deles
que eu vou encarar como pentágonos pois possuem 5 arestas.

Os pentágonos sao:

- O polígono superior esquerdo
- O polígono superior direito
- O polígono inferior central

Imaginemos um destes pentágonos. Chamemos de PS o ponto em que comecamos a
desenhar a suposta curva e PF o ponto em que terminamos de desenhá-la.
Cada vez que a curva cortar uma aresta do pentágono contaremos como 1
CORTE.

Vamos imaginar um contra-exemplo para o enunciado, ou seja, ao menos uma
curva que não passa por qualquer dos vértices e que cruza todas as arestas
APENAS uma vez.

Caso nao haja tal contra-exemplo estará demonstrado que:

Qualquer curva que não passa por qualquer dos vértices mas que cruza todas
as arestas devera cruzar ao menos uma das arestas mais de uma vez.


Há 2 hipóteses:

 a) PS é interior ao pentágono  -- neste caso, após 5 CORTES em arestas
distintas (1 CORTE por aresta), PF tem de ser EXTERIOR ao pentágono;

 b) PS é exterior ao pentágono -- neste caso, após 5 CORTES em arestas
distintas (1 CORTE por aresta),  PF tem de ser INTERIOR ao pentágono;

Ora, o mesmo raciocíno pode ser aplicado aos 2 outros pentágonos.

Agora, verifique que há 2 casos que devemos considerar:

 I) PS é interior ao pentágono superior direito:

 Neste caso, é evidente que PS tem de ser exterior aos 2 outros pentágonos.
PS ser exterior ao pentágono superior esquerdo (hipótese b) faz com que PF
seja interior a ele. Mas PS ser exterior ao pentágono inferior central
(hipótese b) faz com que PF seja também interior a ele. Como PF nao pode ser
interior a 2 pentágonos distintos simultaneamente, chegamos a um ABSURDO.

 II) PS é exterior ao pentágono superior direito:

 Neste caso, pela hipótese b, PF deve ser interior a este pentágono. Assim,
é evidente que PF tem de ser exterior aos 2 outros pentágonos. Mas PF ser
exterior ao pentágono superior esquerdo implica que PS seja interior a ele
(pois caso contrário, pela hipótese b, PF seria interior a este pentágono, o
que é impossível). Pela mesma razao, PF ser exterior ao pentágono inferior
central faz com que PS seja também interior a ele. Como PS nao pode ser
interior a 2 pentágonos distintos simultaneamente, chegamos a um ABSURDO.

Como nao há contra-exemplo para o enunciado que nao nos leve a um absurdo,

CONLUSAO:
Qualquer curva que não passa por qualquer dos vértices mas que cruza todas
as arestas devera cruzar ao menos uma das arestas mais de uma vez.

C.Q.D.

[ ]'s

Alexandre Terezan
-Mensagem Original-
De: Paulo Santa Rita [EMAIL PROTECTED]
Para: [EMAIL PROTECTED]; [EMAIL PROTECTED]
Enviada em: Quarta-feira, 14 de Novembro de 2001 14:52 Terezan
Assunto: Traducao dos Problemas Russos


Ola Pessoal,
Tudo Legal ?

Talvez interesse a alguns estudantes que se preparam para Olimpiadas a
traducao que fiz dos 100 primeiros problemas russos. Coloquei em formato
Word para Windows.

Como nao podemos remeter para esta lista mensagens com arquivos anexados,
quem se interessar em ter estas traducoes basta me enviar um pedido por
e-mail que responderei com as traducoes anexadas.

Acrescento abaixo o primeiro problema :

1) Dados 12 vértices e 16 arestas dispostos como no diagrama abaixo :

X-X-X
| | |
X--X--X--X--X
|  | |  |
X--X-X--X

Prove que qualquer curva que não passa por qualquer dos vértices mas que
cruza todas as arestas devera cruzar ao menos uma das aresta mais de uma
vez.

Um Grande abraco a Todos !
Paulo Santa Rita
4,1251,141101





_
Chegou o novo MSN Explorer. Instale já. É gratuito!
http://explorer.msn.com.br





Re: Traducao dos Problemas Russos

2001-12-04 Por tôpico Alexandre F. Terezan

Olá Paulo e demais integrantes da lista.
Eu nao sei se alguém já respondeu ao problema antes, mas lá vai uma
tentativa.

Gostaria que comentassem, minha solucao é tao elementar que acho q está
errada, hehehe

Imaginemos separadamente cada um dos 5 polígonos delimitados.

2 deles sao verdadeiros retangulos, cada um com 4 arestas. Mas há 3 deles
que eu vou encarar como pentágonos pois possuem 5 arestas.

Os pentágonos sao:

- O polígono superior esquerdo
- O polígono superior direito
- O polígono inferior central

Imaginemos um destes pentágonos. Chamemos de PS o ponto em que comecamos a
desenhar a suposta curva e PF o ponto em que terminamos de desenhá-la.
Cada vez que a curva cortar uma aresta do pentágono contaremos como 1
CORTE.

Vamos imaginar um contra-exemplo para o enunciado, ou seja, ao menos uma
curva que não passa por qualquer dos vértices e que cruza todas as arestas
APENAS uma vez.

Caso nao haja tal contra-exemplo estará demonstrado que:

Qualquer curva que não passa por qualquer dos vértices mas que cruza todas
as arestas devera cruzar ao menos uma das arestas mais de uma vez.


Há 2 hipóteses:

 a) PS é interior ao pentágono  -- neste caso, após 5 CORTES em arestas
distintas (1 CORTE por aresta), PF tem de ser EXTERIOR ao pentágono;

 b) PS é exterior ao pentágono -- neste caso, após 5 CORTES em arestas
distintas (1 CORTE por aresta),  PF tem de ser INTERIOR ao pentágono;

Ora, o mesmo raciocíno pode ser aplicado aos 2 outros pentágonos.

Agora, verifique que há 2 casos que devemos considerar:

 I) PS é interior ao pentágono superior direito:

 Neste caso, é evidente que PS tem de ser exterior aos 2 outros pentágonos.
PS ser exterior ao pentágono superior esquerdo (hipótese b) faz com que PF
seja interior a ele. Mas PS ser exterior ao pentágono inferior central
(hipótese b) faz com que PF seja também interior a ele. Como PF nao pode ser
interior a 2 pentágonos distintos simultaneamente, chegamos a um ABSURDO.

 II) PS é exterior ao pentágono superior direito:

 Neste caso, pela hipótese b, PF deve ser interior a este pentágono. Assim,
é evidente que PF tem de ser exterior aos 2 outros pentágonos. Mas PF ser
exterior ao pentágono superior esquerdo implica que PS seja interior a ele
(pois caso contrário, pela hipótese b, PF seria interior a este pentágono, o
que é impossível). Pela mesma razao, PF ser exterior ao pentágono inferior
central faz com que PS seja também interior a ele. Como PS nao pode ser
interior a 2 pentágonos distintos simultaneamente, chegamos a um ABSURDO.

Como nao há contra-exemplo para o enunciado que nao nos leve a um absurdo,

CONLUSAO:
Qualquer curva que não passa por qualquer dos vértices mas que cruza todas
as arestas devera cruzar ao menos uma das arestas mais de uma vez.

C.Q.D.

[ ]'s

Alexandre Terezan
-Mensagem Original-
De: Paulo Santa Rita [EMAIL PROTECTED]
Para: [EMAIL PROTECTED]; [EMAIL PROTECTED]
Enviada em: Quarta-feira, 14 de Novembro de 2001 14:52 Terezan
Assunto: Traducao dos Problemas Russos


Ola Pessoal,
Tudo Legal ?

Talvez interesse a alguns estudantes que se preparam para Olimpiadas a
traducao que fiz dos 100 primeiros problemas russos. Coloquei em formato
Word para Windows.

Como nao podemos remeter para esta lista mensagens com arquivos anexados,
quem se interessar em ter estas traducoes basta me enviar um pedido por
e-mail que responderei com as traducoes anexadas.

Acrescento abaixo o primeiro problema :

1) Dados 12 vértices e 16 arestas dispostos como no diagrama abaixo :

X-X-X
| | |
X--X--X--X--X
|  | |  |
X--X-X--X

Prove que qualquer curva que não passa por qualquer dos vértices mas que
cruza todas as arestas devera cruzar ao menos uma das aresta mais de uma
vez.

Um Grande abraco a Todos !
Paulo Santa Rita
4,1251,141101





_
Chegou o novo MSN Explorer. Instale já. É gratuito!
http://explorer.msn.com.br





Pergunta intrigante

2001-11-29 Por tôpico Alexandre F. Terezan



 Há pouco tempo um aluno me perguntou sobre 
uma questao do IME 2001, que pedia para demonstrar que (x + y + z)/3 = 
3r(xyz), x0, y0, z0 onde 3r está representando "raiz cúbica de" e 
= o sinal de "maior ou igual a"

 Nós já havíamos trabalhado por alto a 
desigualdade das médias, daí ele me fez a pergunta que eu nao soube 
responder:

 "Ora, sabemos que a média aritmética de n 
termos é maior ou igual à média geométrica destes termos. Como vale para n, vale 
para 3. Resolvido o problema?"

 Minha opiniao PARTICULAR é q 
nao...

 É óbvio que eu nao defendo a teoria de que 
pra usar "Pitágoras" em uma prova temos de antes demonstrá-lo...

 Mas também acho que deve haver bom senso na 
resolucao de uma prova. O que vocês da lista têm a dizer?

 Eu resolveria a questao da seguinte 
maneira:

Seja nr(x) a raiz de índice n do número x.

1) Primeiro provemos que (x+y)/2 = 2r(xy) 
-- (x+y) = 2* 2r(xy) -- (x+y)^2 = 4xy 
--

(x-y)^2 = 0, que é sempre verdadeiro.

Assim, analogamente (z+w)/2 = 
2r(zw) e (c+d)/2 = 2r(cd)

Seja (x+y+z+w)/4 = a.

a = [(x+y)/2 + (z+w)/2]/2 = [2r(xy) + 
2r(zw)]/2

Se c = 2r(xy) e d = 2r(zw), vem:

a = (c+d)/2 = 2r(cd) = 2r[2r(xy) * 2r(zw)] = 
4r(xyzw)

Fazendo w = (x+y+z)/3, vem:

a = [x+y+z + (x+y+z)/3 ]/4 = (x + y + z)/3 = w

Como a = 4r(xyzw),entao:

w = 4r(xyzw) -- w^4 
=xyzw -- w^3 = xyz

Ou:

(x+y+z)/3 = 3r(xyz), c.q.d.






Re: Pergunta intrigante

2001-11-29 Por tôpico Alexandre F. Terezan



Mas aí que está o grande problema...

Se o candidato pudesse assumir um "É fácil ver que" e^x 
= 1 + x, tudo bem...

Mas a meu ver nao pode... teria de provar tal 
desigualdade

  -Mensagem Original- 
  De: Luis Lopes 
  
  Para: [EMAIL PROTECTED] 
  Enviada em: Quinta-feira, 29 de Novembro 
  de 2001 21:00 Terezan
  Assunto: Re: Pergunta intrigante
  
  Sauda,c~oes,
  
  Esta questão teria sido uma ótima ocasião para o candidato 
  ganhar os
  pontos rapidamente.
  
  A demonstração que segue já apareceu numa RPM.
  
  Considere a desigualdade (pulo do gato para este nível) e^x 
  = 1 + x, (*)para
  todo x em R. E e^x = 1 + x == x=0.
  
  Substitua x por a_i/A - 1, com i = 1,2,...,n em (*)e 
  some os n resultados.
  Você chegará a 1 = G^n / A^n 
  ou A = G.
  
  []'s
  Luis
  
  
-Mensagem Original- 
    De: 
    Alexandre F. Terezan 
Para: OBM 
Enviada em: Quinta-feira, 29 de 
Novembro de 2001 17:05
Assunto: Pergunta intrigante

 Há pouco tempo um aluno me perguntou 
sobre uma questao do IME 2001, que pedia para demonstrar que (x + y + z)/3 
= 3r(xyz), x0, y0, z0 onde 3r está representando "raiz 
cúbica de" e = o sinal de "maior ou igual a"

 Nós já havíamos trabalhado por alto a 
desigualdade das médias, daí ele me fez a pergunta que eu nao soube 
responder:

 "Ora, sabemos que a média aritmética de 
n termos é maior ou igual à média geométrica destes termos. Como vale para 
n, vale para 3. Resolvido o problema?"

 Minha opiniao PARTICULAR é q 
nao...

 É óbvio que eu nao defendo a teoria de 
que pra usar "Pitágoras" em uma prova temos de antes 
demonstrá-lo...

 Mas também acho que deve haver bom 
senso na resolucao de uma prova. O que vocês da lista têm a 
dizer?

 Eu resolveria a questao da seguinte 
maneira:

Seja nr(x) a raiz de índice n do número x.

1) Primeiro provemos que (x+y)/2 = 2r(xy) 
-- (x+y) = 2* 2r(xy) -- (x+y)^2 = 4xy 
--

(x-y)^2 = 0, que é sempre verdadeiro.

Assim, analogamente (z+w)/2 = 
2r(zw) e (c+d)/2 = 
2r(cd)

Seja (x+y+z+w)/4 = a.

a = [(x+y)/2 + (z+w)/2]/2 = [2r(xy) + 
2r(zw)]/2

Se c = 2r(xy) e d = 2r(zw), vem:

a = (c+d)/2 = 2r(cd) = 2r[2r(xy) * 2r(zw)] = 
4r(xyzw)

Fazendo w = (x+y+z)/3, vem:

a = [x+y+z + (x+y+z)/3 ]/4 = (x + y + z)/3 = 
w

Como a = 4r(xyzw),entao:

w = 4r(xyzw) -- w^4 
=xyzw -- w^3 = xyz

Ou:

(x+y+z)/3 = 3r(xyz), c.q.d.






Re: 4 Questoes

2001-11-27 Por tôpico Alexandre F. Terezan

Aí está o enunciado correto do problema 4:

4)Seja um paralelogramo ABCD. Traça-se uma reta que passa por D e corta
o lado BC no ponto P e o prolongamento do lado AB no ponto Q. Se a área do
triângulo DPC vale 8 e a área do quadrilátero ABPD vale 29, quanto vale
a área do triângulo CPQ?



-Mensagem Original-
De: [EMAIL PROTECTED]
Para: [EMAIL PROTECTED]
Enviada em: Segunda-feira, 26 de Novembro de 2001 23:50 Terezan
Assunto: Re: 4 Questoes


4) Seja um paralelogramo ABCD. Traça-se uma reta que passa por D e corta
o lado BC no ponto P e o prolongamento do lado AB no ponto Q. Se a área
do
triângulo DAC vale 8 e a área do quadrilátero ABCD vale 29, quanto vale
a
área do triângulo CPQ?

Se a área de DAC=8, então a de ABCD será 16, e não 29. 
Não será a área de ACPQ=29??

[]'s, Yuri
ICQ: 64992515


--
Use o melhor sistema de busca da Internet
Radar UOL - http://www.radaruol.com.br







Re: Um tal de Newton...

2001-11-21 Por tôpico Alexandre F. Terezan

Uma pequena distracao:

(1 + 3x + 2x^2) = 2(x+1)(x+1/2)
e nao
(1 + 3x + 2x^2) = (x+1)(x+1/2)

-Mensagem Original-
De: Alexandre Tessarollo [EMAIL PROTECTED]
Para: [EMAIL PROTECTED]
Enviada em: Quarta-feira, 21 de Novembro de 2001 02:41 Terezan
Assunto: Re: Um tal de Newton...




[EMAIL PROTECTED] wrote:

 Meus cumprimentos,

 Estava estudando um tal de Newton e encontrei uma questão
 interessante, embora eu esteja errando algo simples pra vocês...

 Questão (FFCLUSP)
 Mostrar que o coeficiente de x^8 no desenvolvimento
 de (1 + 3x + 2x^2)^10 é 3780.

 Meu erro: os coeficientes de x e de x^2 estão fazendo o
 coeficiente do termo x^8 ficar muito grande ...

Vale lembrar q se vc estiver tentando usar a fórmula do Binômio de
Newton, ela só vale para BInômios, ou seja, algo como (x+a)^n. Nós temos
um TRInômio...

Bem, mas vamos tentar... Sabemos q o trinômio pode ser reescrito como
(x+1)(x+1/2). Assim, queremos saber o coeficiente de x^8 no
desenvolvimento de
[(x+1)(x+1/2)]^10= [(x+1)^10][(x+1/2)^10]

Seja a[i]x^i o termo de grau i do primeiro binômio e, p/não
confunidir as letras, a[j]x^j o de grau j do segundo binômio. Assim, o
nosso polinômio final terá termos da forma a[i]a[j]x^(i+j), com i e
j variando (independentemente) de 0 a 10.

Dessa forma, temos que achar i+j=8. As soluções (i;j) que estão no
nosso intervalo são: (0;8), (1;7), (2;6), (3;5), (4;4), (5;3), (6;2),
(7;1) e (8;0). Agora sim podemos utilizar a fórmula do Binômio de
Newton, calcular os coeficientes com i e j das soluções, fazer as
devidas contas e pronto. Sei q deve dar algum trabalho, mas depois posso
até fazer caso alguém queira. Como a essa hora meus neurônios já foram
dormir, fico devendo uma solução mais concisa e prática.

[]'s

Alexandre Tessarollo

 Caso alguém queira tentar...

 Muito grato,

 Héduin Ravell

 _
 Do You Yahoo!?
 Get your free @yahoo.com address at http://mail.yahoo.com





Re: IME (era: Re:dúvida)

2001-11-14 Por tôpico Alexandre F. Terezan

Na verdade é possível resolver para o caso geral  sqrt(a-sqrt(a-x))=x
sabendo-se que x0.

Essa foi a segunda maneira que eu, particularmente, enxerguei...

a primeira foi a de aplicar infinitas vezes f(x) = sqrt(5-x), que pra mim
foi a mais imediata...

 Voltando ao caso geral, a idéia é resolver a equacao de segundo grau em
a...

Essa nao foi a primeira e nem será a última vez que se resolve uma equacao
em x por um artifício desses... eu já tinha utilizado este artifício para
uma equacao MUITÍSSIMO parecida com este caso geral...

Dentre as 4 respostas obtidas para x, apenas uma é a correta...

Se alguém desejar, eu mostro em detalhes, mas nao creio q seja necessário...

-Mensagem Original-
De: Alexandre Tessarollo [EMAIL PROTECTED]
Para: [EMAIL PROTECTED]
Enviada em: Quarta-feira, 14 de Novembro de 2001 19:17 Terezan
Assunto: IME (era: Re:dúvida)




luis felipe wrote:

 concordo com o alexandre

 a prova do IME deste ano foi bem elaborada, embora eu ache que duas
questões
 estavem pesadas demais para alunos de 2 grau( 7 e a 9) devemos lamentar
 também uma falha grave no enunciado da questão 8

 valeu

 luis felipe

Já que ninguém comenta, comento eu. Comecemos pela questão 9.

Resolva a equação sqrt(5-sqrt(5-x))=x sabendo-se que x0.

Eu já devo ter visto umas 4 soluções diferentes, mas em quase todas
havia
pelo menos um passo não justificado ou questionável... Uma delas era:

Seja f(x) = sqrt(5-x). Temos f(f(x))=x. Logo,  f(x)=f^(-1)(x). Aí vem a
parte é
fácil ver que os gráficos de f(x) e de f^(-1)(x) se cruzam sobre a reta
y=x. A
partir daí, temos f(x)=x, resolve-se uma equação do segundo grau e pronto.
Mas
falta demonstrar a parte é fácil ver...

Outra diz:

Aplicando f(x) nela mesma 2n vezes, com n tendendo ao infinito, teremos
f(f(f(f((f(x))...=x. Logo, podemos trocar todos os
f(f(f...(f(x))...)))
de dentro do primeiro f por x. Assim teremos f(x)=x e novamente é só
resolver a eq do segundo grau. A solução, olhando com carinho, está certa,
mas
foi utilizado o conceito de limite.

Ainda há uma terceira, esta já sem erros mas um pouco mais longa. Trata-se
da
solução do Poliedro:

Como x0 e real, temos que 0x5. Tome y=sqrt(5-x) (I). A equação
original
transforma-se em
sqrt(5-y)=x (II)
Elevando I e II ao quadrado, temos:
y^2=5-xIII
x^2=5-yIV
Fazendo III-IV, temos
y^2-x^2=y-x
(y+x)(y-x)=y-x
(y+x)(y-x)-(y-x)=0
(y-x)(y+x-1)=0

Segue que
y-x=0V

OU

y+x-1=0VI

De V segue a nossa equação do segundo grau. Considerando o intervalo
0x5,
só teremos uma resposta - a certa. Falta examinar VI. Substituindo-a em III
ou
IV, teremos uma equação do segundo grau que resulta só uma resposta no
intervalo
0x5. Contudo, como elevamos ao quadrado as eqs I e II p/chegarmos a III e
IV,
precisamos verificar via teste se essas duas soluções servem ou não. Fazendo
isso só teremos a resposta correta...

Há ainda uma resposta, esta feita pelo Prof. Raul Agostino:

Elevando a equação ao quadrado e arrumando, temos:
5-x=sqrt(5-x)

Elevando novamente, temos:

25-10x^2+x^2=5-x

O que todo mundo tenta daqui em diante é somar tudo num lado só, chegar
num
polinômio do QUARTO grau e não conseguir resolvê-lo - ele não possui raízes
óbvias, sequer inteiras... O pulo do gato segue abaixo, se vc não quiser
ver,
pare aqui..

10

9

8

7

6

5

4

3

2

1

0

Bum!! Brincadeirinha... :0)

Olhando com MUITO carinho e MUITA boa vontade, podemos arrumar a equação
assim:

25-(2x^2+1)5+x^4+x=0

Um olho treinado verá uma equação do SEGUNDO grau em CINCO. Isso mesmo,
algo
da forma a(5^2)+b(5)+c=0. Resolvendo, teremos:

5=(2x^2+1 +-sqrt(4x^4+4x^2+1-4x^4-4x))/2
Dentro da raiz fica 4x^2+1-4x = (2x-1)^2. Tirando a raiz, deveríamos colocar
o
módulo mas, como já existe o +-, basta colocar direto mesmo. Fica:

5=(2x^2+1 +-(2x-1))/2

Resolvendo e respeitando os intervalos, teremos a solução...

[]'s

Alexandre Tessarollo





Re: Definicoes Urgentes (para hoje)

2001-11-01 Por tôpico Alexandre F. Terezan

Muito Obrigado, Alvaro e Haroldo, pela ajuda!!!

- Original Message -
From: Alvaro de Jesus Netto [EMAIL PROTECTED]
To: [EMAIL PROTECTED]
Sent: Quinta-feira, 1 de Novembro de 2001 02:45 Terezan
Subject: Re: Definicoes Urgentes (para hoje)


Saudacoes a todos.

1) Matriz adjunta é a transposta da matriz dos cofatores.

Os cofatores sao construidos da seguinte forma:
A(i,j) = ((-1) ^ (i + j)) * Det(B), onde B eh a matriz resultante da
retirada
da i-esima linha e j-esima coluna da matriz original.

A matriz adjunta eh muito util para calculo da inversa.

2) Um cone sera equilatero se for reto e sua secao transversal for um
triangulo equilatero.

Abracos, Alvaro.

  Saudacoes...

 Por favor, alguém poderia me ajudar com estas duas definicoes simples?

 1) O que é A*, a matriz adjunta de A, e como se calcula?

 2) Qual a condicao para definirmos um CONE como equilátero?


Content-Type: text/html; charset=iso-8859-1; name=Anexo: 1
Content-Transfer-Encoding: quoted-printable
Content-Description:


--
Nome: Alvaro de Jesus Netto.
e-mail: [EMAIL PROTECTED] (atenção para o zero após o r).





Re: Outro probleminha sobre horas.

2001-10-31 Por tôpico Alexandre F. Terezan

VOU CONSIDERAR COMO Ângulo formado entre os outros dois como o ÂNGULO
AGUDO FORMADO ENTRE OS OUTROS DOIS.

Caso eu tenha compreendido mal, a resposta estará incorreta, mas o
raciocínio será o mesmo.

Seja Ag a representacao de A graus

No caso dos ponteiros das horas, 1h = 30g, 1 min = 0,5g e 1s = 1/120g

No caso dos ponteiros dos minutos, 1 min = 6g e 1s = 0,1g

No caso dos ponteiros dos segundos, 1s = 6g

Verifique agora que de 12h00min00s às 12h01min00s o ponteiro dos segundos
NUNCA se encontra entre os outros dois.

No entanto, entre 12h01min00s e 12h02min00s obrigatoriamente o ponteiro dos
segundos se torna bissetriz dos outros dois...

Assim, o horário pedido é  12h01minXs

Devemos encontrar tal X.

Seja B(h) o ângulo formado entre o ponteiro das horas e a marca 12 do
relógio, B(min) o ângulo formado entre o ponteiro dos minutos e a marca 12
do relógio e B(s) o ângulo formado entre o ponteiro dos segundos e a marca
12 do relógio.

Queremos que B(min) - B(h) = 2 * [ B(s) - B(h) ]

Mas B(h) = 1/2 + X/120
B(min) = 6 + X/10
B(s) = 6X

Resolvendo, vem X ~= 0,5466 segundos

O horário pedido é 12h01min e 0,5466 segundos.

PS: Se for pedido a bissetriz de qualquer ângulo formado, entao a bisseteriz
se dará logo no primeiro minuto, em torno de:
12h00min30,5segundos (PROVE ISTO)


- Original Message -
From: Alvaro de Jesus Netto [EMAIL PROTECTED]
To: [EMAIL PROTECTED]
Sent: Quarta-feira, 31 de Outubro de 2001 12:48 Terezan
Subject: Outro probleminha sobre horas.


Saudacoes aos colegas da lista.

Ao meio dia os ponteiros de um relogio (hora, minuto e segundo) estao
superpostos. Quando, apos essa superposicao, pela primeira vez, o ponteiro
dos segundos sera bissetriz do angulo formado pelos outros dois?

Abracos, Alvaro.
--
Nome: Alvaro de Jesus Netto.
e-mail: [EMAIL PROTECTED] (atenção para o zero após o r).





Definicoes Urgentes (para hoje)

2001-10-31 Por tôpico Alexandre F. Terezan



Saudacoes...

Por favor, alguém poderia me ajudar com estas duas definicoes 
simples?

1) O que é A*, a matriz adjunta de A, e como se calcula? 


2) Qual a condicao para definirmos um CONE como 
equilátero?


Re: Minimo

2001-10-19 Por tôpico Alexandre F. Terezan

Gostaria que vcs verificassem se minha resposta está CORRETA, uma vez q nao
me propus a utilizar derivadas...

Seja f(x) = x^x , para x real positivo...

Se k é também um real positivo, entao f(x+k) = (x+k)^(x+k)

Ora, para que f(x+k)  f(x), entao: (x+k)^(x+k)  x^x

Entao: [(x+k)/x]^x  1/[(x+k)^k]

Mas como sabemos que [(x+k)/x]^x = [1 + (k/x)]^x  e^k para todo k e x real,
entao:

1/[(x+k)^k]   e^k , ou  x+k  1/e  --  x  1/e - k

Como isto vale para TODO k real positivo, entao NECESSARIAMENTE:

x  1/e   ou   x = 1/e.   Ou, seja, para todo x = 1/e  a funcao f(x) é
crescente.

Por outro lado, para que f(x-k)  f(x), entao (x-k)^(x-k)  x^x

Logo, [(x-k)/x]^x  (x-k)^k   . Como [(x-k)/x]^x = [1 - (k/x)]^x, que é
menor que [(1/e)^k] para todo k e x reais positivos, entao:

(x-k)^k[(1/e)^k]--x-k  1/e--  x  1/e + k

Como isto vale para TODO k real positivo, entao NECESSARIAMENTE:

x  1/e  ou  x = 1/e .  Ou, seja, para todo x = 1/e  a funcao f(x) é
decrescente.

Mas se f(x) é decrescente até x = 1/e e é crescente a partir deste valor,
entao o valor MÍNIMO de f(x) é obrigatoriamente f(1/e) = (1/e)^(1/e).

Correto?







- Original Message -
From: Nicolau C. Saldanha [EMAIL PROTECTED]
To: [EMAIL PROTECTED]
Sent: Sexta-feira, 19 de Outubro de 2001 06:57 Terezan
Subject: Re: Minimo


On Thu, Oct 18, 2001 at 09:50:40PM -0200, Anselmo Alves de Sousa wrote:
 Olah! Mais uma vez venho aqui com uma duhvida. E quem diria? O professor
...
 Legal. Aplicando a primeira derivada resolvemos rapidamente. O problema é
 que a resposta foi dada na forma de raiz de indice e de 1/e.

 Gostaria de saber se existe significado para raizes de indice irracionais.
 Outro exemplo: raiz de indice pi.

A explicação mais simples e elementar que eu conheço é a seguinte.

LEMA:
Dado um número real a  1 existe uma única função crescente f: R - R
satisfazendo f(1) = a e f(x+y) = f(x) f(y).
Dado um número real 0  a  1 existe uma única função crescente f: R - R
satisfazendo f(1) = a e f(x+y) = f(x) f(y).

Aceitando este lema definimos a^x = f(x).
A raiz de índice x de a é f(1/x).
Por alguma razão esta apresentação parece ser incomum.
Parece que a maioria dos autores acha que é necessário
falar de limites para definir a^x.
Na pior das hipóteses é necessário usar conceitos de sofisticação
comparável ao de limite para *provar* o lema, mas não para definir.

[]s, N.






Re: Problema bonito

2001-10-11 Por tôpico Alexandre F. Terezan



Sério, como?

  - Original Message - 
  From: 
  harold 
  
  To: [EMAIL PROTECTED] 
  Sent: Quinta-feira, 11 de Outubro de 2001 
  04:13 Terezan
  Subject: Re: Problema bonito
  
  
  
-Mensagem original-De: 
Alexandre F. Terezan [EMAIL PROTECTED]Para: 
OBM [EMAIL PROTECTED]Data: 
Quarta-feira, 10 de Outubro de 2001 15:51Assunto: Problema 
bonito
Seja 7@30' a representacao de 7 graus e 30 
minutos.
Seja sqrtx a raiz quadrada de x


Demonstre que tg ( 7@30 )= sqrt6 - sqrt3 + sqrt2 - 
2



sabemos que tg 2a= 2tga/1-tga .tga

e sabemos que tg 15= 2 - sqrt3 desenvolvemos a fórmula 
acima e chegamos a uma equação de grau 2.encontrando o resultado 
acima.


Re: Problema sobre primos

2001-10-08 Por tôpico Alexandre F. Terezan

Tem certeza de q vc escreveu corretamente a funcao???


- Original Message -
From: Paulo Jose Rodrigues [EMAIL PROTECTED]
To: [EMAIL PROTECTED]
Sent: Segunda-feira, 8 de Outubro de 2001 16:15 Terezan
Subject: Re: Problema sobre primos


 Se vc resolveu esse problema, vc deveria dar conferencias no mundo
 inteirovc seria maior que Gauss ou EinsteinTal formula não
existe!!!
 Um abraço e  não se engane...essa lista não tem ingênuos...
Ruy

Ruy,
 existem infinitas fórmulas que geram somente números primos. O que acontece
é que a grande maioria dessas fórmulas são inúteis do ponto de vista
prático.

Por exemplo, não é difícil provar com a ajuda do Teorema de Wilson, que a
função

f(x, y)=(y-1)/2[|B^2-1|-(B^2-1)]+2,

onde B=x(y+1)-(y!+1), x e y são números naturais, gera somente números
primos, gera todos os primos e gera todos os primos ímpares exatamente uma
vez.







equacao

2001-09-13 Por tôpico Alexandre F. Terezan



Estou empacado numa equacao... Parece ser óbvio, mas nao 
consigo enxergar...

a^2 + b^2 = 16
a^2 + (c - b)^2 = 25
b^2 + (c - a)^2 = 1

Quanto vale c^2 ???


Re: Encontrar os números inteiros

2001-08-30 Por tôpico Alexandre F. Terezan



Dividindo (n^2 + 7) por (n+3) encontramos quociente (n-3) e 
resto 16.

Ou seja, (n^2 + 7) = (n+3)(n-3) + 16

Logo, (n^2 + 7)/(n+3) = (n-3) + 
16/(n+3), que é inteiro quando (n+3) divide 
16.

Divisores inteiros de 16: -16, -8, -4 , -2, -1, 1, 2, 4, 8, 
16

(n+3) = -16 -- n = 
-19

(n+3) = -8 -- n = 
-11

(n+3) = -4  -- n = 
-7

(n+3) = -2  -- n = 
-5

(n+3) = -1  -- n = 
-4

(n+3) = 1  -- n = 
-2

(n+3) = 2 -- n = 
-1

(n+3) = 4 -- 
n = 1

(n+3) = 8 -- n = 
5

(n+3) = 16 -- n = 
13

- Original Message - 


  From: 
  Davidson 
  Estanislau 
  To: obm 
  Sent: Quinta-feira, 30 de Agosto de 2001 
  08:08 Terezan
  Subject: Encontrar os números 
  inteiros
  
  
  
   Encontrar todos os números inteiros n tais que (n^2 + 
  7)/(n + 3) também é um número inteiro.
  
   
  Davidson


Re: Estatística e resta -um

2001-08-19 Por tôpico Alexandre F. Terezan

A questao do resta-um nao é difícil, se vc já viu a resposta antes... :-)

A verdade é que a dificuldade desta questao (assim como de muitas outras)
reside na elaboracao de como atacá-la...

Bom, desenharei, na medida do possível, o tabuleiro de resta-um, com as
casas inicialmente OCUPADAS pelas cores A,B,C apropriadamente.

C A B
B C A
B C A B C A B
A B C B C A
C A B C A B C
A B C
C A B

Repare que, o primeiro movimento significa inevitavelmente apagarmos 1 cor
B e 1 cor C e pintarmos uma cor A no centro.

Analogamente, todos os movimentos se resumem em 3:
- apagar 1B e 1C, pintar 1A
- apagar 1A e 1C, pintar 1B
- apagar 1A e 1B, pintar 1C

Assim, em todos os casos, adicionamos ou reduzimos 1 unidade do total de
unidades de determinada cor.
Seja x{n} o número de casas pintadas da cor X após a jogada n.

Logo, a{0} = 10, b{0} = 11 , c{0} = 11.

Consideremos a soma a{n} + b{n}.
Ora, a{n+1} = a{n} +- 1
b{n+1} = b{n} +- 1

Logo, a{n+1} + b{n+1} = a{n} + b{n} + {-2,0,2}
Assim, conclui-se que (a{n+1} + b{n+1}) possui a mesma paridade de (a{n} +
b{n}), que possui a mesma paridade de (a{0} + b{0}), q é impar (10+11=21).

Analogamente, conclui-se que a{n} + c{n} é ímpar, já que   a{0} + c{0} = 21,
e que b{n} + c{n} é par, visto que b{0} +c{0} = 22.

Seja k a última rodada do jogo.
- Se a última peça estiver numa casa B, a{k} + c{k} = 0, impossível, pois
a{k} + c{k} é ímpar.
- Se a última peça estiver numa casa C, a{k} + b{k} = 0, impossível, pois
a{k} + b{k} é ímpar.
- Se a última peça estiver numa casa A, a{k} + c{k} = 1, a{k} + b{k} = 1
e b{k} + c{k} = 0, o que é possível.

Logo, obrigatoriamente a última peça deve estar em uma casa A.

Se pintarmos o tabuleiro da maneira oposta:

   C A B
   A B C
C A B C A B C
A B C B C A
B C A B C A B
B C A
C A B

Concluímos novamente que a última peça só pode repousar em A.

Somando os dois casos, as únicas casas que sao de cor A em ambas as maneiras
de pintar sao 5:

- a casa central
- a casa da 1a fileira, 4a coluna
- a casa da 4a, fileira, 1a coluna
- a casa da 7a fileira, 4a coluna
- a casa da 4a fileira, 7a coluna

Logo, só estas casas podem abrigar o último pino.

- Original Message -
From: [EMAIL PROTECTED]
To: [EMAIL PROTECTED]
Sent: Sábado, 18 de Agosto de 2001 13:03 Terezan
Subject: Estatística e resta -um


Caros amigos da lista,
Pode ser que a pergunta seja um pouco off topic , ne verdade, acho que é
completamente off topic, bom, mas já que esou escrevendo o e-mail vamos a
questao.

em fato, nunca entendi muito bem pq o desvio padrao amostral é considerado
mais apurado do que o desvio médio, pelo menos uma razao matematica,
demonstrável, para isso. Será que alguem poderia me ajudar com isso?

Pra nao perder a viagem, aí vai um probleminha (alguém me disse que esta foi
do professor Nicolau)
No jogo resta-um , quando se deixa apenas um pino sobre o tabuleiro, esse
pino só pode ocupar determinadas posiçoes. Determine quantas sao essas
posiçoes.

um abraço a todos,

Thiago Brando









dúvidas banais

2001-05-29 Por tôpico Alexandre F. Terezan



Estou com 2 dúvidas banais, mas que nao consigo solucioná-las 
por nao ter a possibilidade de consultar um livro do ensino médio. Sao 
elas:

O que define uma hipérbole EQUILÁTERA?

O que define um cilindro 
EQUILÁTERO?


mais uma banal

2001-05-29 Por tôpico Alexandre F. Terezan



Mais uma coisinha...

Quais sao os eixos da 
HIPÉRBOLE?


Re: problema de probabilidade...

2001-05-24 Por tôpico Alexandre F. Terezan




Eu encontrei outra resposta para a questao, embora utilizando o mesmo 
raciocínio...

p(31) = (2^30 + 2)/(3 * 2^30) , que tb é próximo de 
1/3.

Da mesma forma, p(7) = 11/32 = (2^6 + 2)/(3 * 2^6).

Para todo n ímpar,p(n) = [2^(n-1)+2]/[3 * 
2^(n-1)]

Para todo n par, p(n) = [2^(n-2)-1]/[3 * 
2^(n-2)]

- Original Message - 
From: "Nicolau C. Saldanha" [EMAIL PROTECTED]
To: [EMAIL PROTECTED]
Sent: Quarta-feira, 23 de Maio de 2001 10:47 Terezan
Subject: Re: problema de probabilidade...
Este problema já caiu em uma OBM, exceto que lá as cores 
erammagenta, amarelo e ciano. Obviamente alguém preferiu trocarpor cores 
mais 'fáceis' sem alterar a primeira letra do nome de cada cor.Até o nome do 
personagem era o mesmo. E quem propôs o problema fui eu.No dia 1 a 
probabilidade dele usar o par de cor M é 1.No dia 2 é 0, no dia 3 é 
1/2.Em geral, se no dia n a probabilidade é p(n), no dia n+1 
seráp(n+1) = (1-p(n))/2. Resolvendo esta recorrência temosp(n) = (1 - 
(-1/2)^(n-2))/3 ep(31) = (1 - (-1/2)^29)/3 = (2^29 - 
1)/(3*2^29)O que está, como era de se esperar, muito perto de 
1/3.[]s, N.On Tue, 22 May 2001, Luis Lopes wrote: 
Sauda,c~oes,  Repasso um problema de uma outra lista. 
 [ ]'s Lu'is   From: "Daniel Cid 
(sinistrow)" [EMAIL PROTECTED] 
Reply-To: [EMAIL PROTECTED] 
To: [EMAIL PROTECTED] 
Subject: [Olympium] problema de probabilidade... Date: Fri, 18 
May 2001 13:31:27 -0300   Alguem 
pode me ajudar nesse problema ??  Jose tem tres pares de 
oculos, um marrom, um amarelo e um cinza. Todo dia ele escolhe um ao 
acaso, tendo apenas o cuidado de nunca usar o mesmo que usou no dia 
anterior. Se dia primeiro de agosto ele usou o marrom. Qual a 
probabilidade de que no dia 31 de agosto ele volte a usar o marrom ??? 
 []`z  -- Daniel B. Cid 
(sinistrow)  


Re: Dois problemas de Teoria dos Números.

2001-05-10 Por tôpico Alexandre F. Terezan




Resposta da Questao 1:

Para K = 1, basta escolhermos um M composto qualquer (10, por 
exemplo).
Para K  1, basta fazer M = [(K+1)! + 2]

- Original Message - 
From: "Marcos Eike" [EMAIL PROTECTED]
To: [EMAIL PROTECTED]
Sent: Quinta-feira, 10 de Maio de 2001 22:56
Subject: Dois problemas de Teoria dos Números.
Pessoal, vcs poderiam fornecer soluções interessantes 
para:1) Let K be a positive integer. Prove that the sequence of natural 
numberscontains an infinite set of sequence M, M+1, ..., M+K-1, not 
containingprimes.2)Prove that there an infinite numbers composite 
among the numbersrepresented by the polynomial a_0 * x^n + a_1 * x^(n-1) + 
... + a_n, wherea_0, a_1, ... , a_n are integer and a_0  0.Por 
favor!!!Ats,Marcos Eike


Re: Sobre o Problema 3N+1

2001-05-09 Por tôpico Alexandre F. Terezan

Seria interessante que vcs compartilhassem idéias e descobertas na lista,
para que possamos todos contribuir...

- Original Message -
From: Paulo Santa Rita [EMAIL PROTECTED]
To: [EMAIL PROTECTED]
Sent: Quarta-feira, 9 de Maio de 2001 12:45
Subject: Re: Sobre o Problema 3N+1


Ola Rui e Colegas da Lista,
Tudo Legal ?

Eu avancei bastante na compreensao deste problema, desde que o Prof Nicolau
o apresentou. Mas desde entao não me ocupei mais com ele. Se voce quiser,
nos podemos trabalhar nele juntos.

Consegui o seguinte :

1) Mapear todos os numeros que com certeza atendem a conjetura, associando a
cada um uma sequencia finita de numeros naturais.
2) Para cada sequencia, conseguo determinar o expoente ^que faz com que
S^p(N)=1
3) Associar a este mapeamente uma rede bastante complicada.

Aqui eu parei.

Minha intuicao :

Se existe um numero tal que não existe p com S^p(N)=1, isto implica que as
sucessivas aplçicaçoes de S conduzirao a uma sequencia infinita. A ideia e
mostrar que isto e impossivel.

Como fazer esta prova :

Estudando as propriedades topologicas da rede ( voce chama de arvore ).



Eu terminei me desinteressando pela questao, pois me envolvi com outras
temas tambem emocionantes ( acredito que descobri as colunas ocultas do
triangulo de Pascal, o que me permite falar em sequencias aritmeticas de
ordem racional. Isto esta diretamente ligado a serie de euler :

1  +  1/4  +   1/9 + ... = (pi)^2/6

agora entendo que a formulacao correta - Tio Euler nao viu isso - e :

1  +  1/4  +  1/9  +  ... = (1/3!)*(1 - 1/3 + 1/5 ...)*(1  -  1/3  +  1/5
... ). É o teorema das colunas generalizado.

posso portanto pensar em encontrar o valor de :

1  +  1/2^r  +  1/3^r  + ...

A partir daqui surge a funcao :

F(r) = 1  +  1/2^r  +  1/3^r  + ...

Ora, esta funcao e um plano vertical cortando a funcao mais geral :
F(z) = 1  +  1/2^z  +  1/3^z  + ...

E isto esta ligado a Conjectura de Riemnam. )

Voce deve ser novo na Lista. Nao me lembro de nenhuma mensagem sua
anteriormente. Se assim for, seja bem vindo.

Eu sou abandonante ( realmente : abandonante = abandonando ) de Engenharia
migrando para Matematica. Se voce quer discutir Matematica, sem frescura e
estrelismos, vai ser legal a nossa correspondencia.

Um Grande abraco pra voce
Paulo Santa Rita
4,0944,09052001


From: Rui Viana [EMAIL PROTECTED]
Reply-To: [EMAIL PROTECTED]
To: [EMAIL PROTECTED]
Subject: Re: Sobre o Problema 3N+1
Date: Tue, 08 May 2001 19:43:25 -0300

Oi Paulo,

Eu soh queria dizer que esse problema do 3N+1 eh um dos que mais me fascina
na matematica. Assim como o ultimo teorema de Fermat, ele tem uma
formulacao
bem simples e ainda estah em aberto. A diferenca eh que esse problema naum
eh tao famoso quanto o de Fermat e eh isso que me fascina nele.
Eu realmente naum sei quais as implicacoes matematicas de uma possivel
solucao ou contra-prova, mas ainda assim de vez em quando eu dou uma
pensada
nele.

A sua ideia eh bem natural , e faz sentido. Resta saber quao dificil saum
as
demonstracoes do buraco. Um outro jeito de olhar eh contruindo uma arvore
que comeca no 1 e vai descendo assim :
1
2
4
8
16
   325
   6410
.

Dai tentar achar algum padrao na posicao de cada numero. sei lah...


Seria muito legal se a lista se envolvesse nesse problema, apresentando
material relativo ao problema, ideias, solucoes..

[]'s
Rui Viana


From: Paulo Santa Rita [EMAIL PROTECTED]
Reply-To: [EMAIL PROTECTED]
To: [EMAIL PROTECTED]
Subject: Sobre o Problema 3N+1
Date: Mon, 07 May 2001 14:10:47

Ola Pessoal !

Pelo que me lembro, o problema 3N+1 foi apresentado a esta lista pelo
Prof
Nicolau. Este problema tambem e conhecido como problema de Siracura,
dentre outras designacoes. Ele pode ser enunciado como segue :

Seja F:N - N uma funcao, tal que
F(n) = 3n+1, se n e impar
F(n) = n/2, se n e par.

Se definirmos : F^p(n)=F(F(F(F(...(p)..., isto e, F^p(n) e a
composicao
de F com ela mesma p vezes, entao :

CONJECTURA : Para todo n natural, existe um p natural tal que
F^p(n)=1.

Este conjectura, pelo que sei, esta em aberto. Muitos Matematicos de
Escol
tentaram prova-la, sem sucesso. Claramente que isso nao significa que
qualquer um de nos tambem nao tera sucesso ...

Aqui nos DISCUTIMOS PROBLEMAS. Nao significa que sempre precisamos
apresentar uma solucao pronta. Podemos inicia-la, podemos clarificar
alguns
aspectos ou apenas apresentar ideias : e a discussao !

O problema acima leva-nos a lembrar dos BLACK HOLE ( Buraco Negro ) ou
SORVEDOUROS ... Com efeito, se para algum n impar aplicarmos F(n)=3n+1 e
o
resultado por uma potencia de 2, entao a ulterior aplicacao de F(n)=n/2
ira
nos conduzir fatalmente a 1. Isto mostra que a sequencia
2,4,8,16,...,2^p,... funciona como um BLACK HOLE  ou SORVEDOURO, de forma
que podemos refornular a conjectura da seguinte maneira :

CONJECTURA1 : Para todo n natural, existe um p 

Re: ITA

2001-05-04 Por tôpico Alexandre F. Terezan



Mais uma coisa...

Eu fui um caso à parte,pois estava em dúvida entre Engenharia 
e Medicina, daí nao ter feito turma IME-ITA, mas sim uma Especial para 
Biomédicas...

Bom, eu nao fiz vestibular pro ITA, mas fui aprovado no IME 
(nao saía classificacao na época)...

Quanto à UFRJ, o que eu tenho a dizer é que com um pouco de 
astúcia boas colocacoes sao bastante plausíveis...

Acredito que minhas notas (que expusno último e-mail) 
foram bem dentro da realidade... nem um pouco excepcionais...

No entanto, eu fiquei em 1olugar no vestibular de 
Medicina, simplesmente porque soube distribuir meus esforços...

Se eu tivesse tirado 8,0 em todas as provas (tática da 
dispersao de esforços) eu até passaria (40,0 pontos) mas minha colocacao 
seriabem inferior...

O que eu quero dizer é que eu só tirei 4 notas acima de 8 no 
Vestibular e passei com bastante folga...

Pra bom entendendor, meia palavra basta... :-)

  - Original Message - 
  From: 
  Fábio Arruda de Lima 
  To: [EMAIL PROTECTED] 
  Sent: Sexta-feira, 4 de Maio de 2001 
  08:22
  Subject: ITA
  
  Caro Gustavo,
  Se realmente é isto que você quer, dar-te-ei 
  algumas dicas. Inicialmente, entre numa turma preparatória IME/ITA. Esta não é 
  a solução, porém vai te ajudar muito.
  Dedique-se as matérias do concurso, estudando 
  todo o programa de matérias. Não deixe de estudar nada, pois uma questão fácil 
  de um assunto não estudado, torna-se muito difícil. Se forem difíceis as 
  questões do assunto que você estudou, teremos um grande problema. 
  Nãoabandone matérias como português e línguas (deixei de ser 1º colocado 
  no IME por causa de português - fui 2º). Tente estudar o maior número de horas 
  possível. Resolva todas as provas anteriores, elas dão umaboa 
  preparação. Faça um estudo sólido e consistente. Decorar física, química ou 
  matemática não é um bom negócio. Tente entender os conceitos e deduzir por si 
  mesmo as fórmulas. E tenha sempre uma coisa em mente, se o Fábio Arruda passou 
  então eu passo (consegui muitas coisas pensando assim - se alguém passou, 
  então eu posso passar). Nada na vida é impossível. Tudo é uma questão de 
  escolha e sacrifícios. Se você sacrificar outros afazeres, certamente você 
  terá tempo suficiente.
  Espero ter ajudado. Um abraço
  Fábio Arruda
  
  
  
  
  
  
- Original Message - 
From: 
Gustavo 
Martins 
To: [EMAIL PROTECTED] 
Sent: Friday, May 04, 2001 1:13 
PM
Subject: Re: Aprendendo mat. sem perder 
o resto

Eu quero é Eng. Aer. no ITAmesmo. Se for 
realmente impossível isso, eu faço física.

[]s
Gustavo

  - Original Message - 
  From: 
  Fábio Arruda de Lima 
  To: [EMAIL PROTECTED] 
  Sent: Thursday, May 03, 2001 1:11 
  PM
  Subject: Re: Aprendendo mat. sem 
  perder o resto
  
  Bom Gustavo,
  Tudo depende do seu objetivo. Quando eu 
  terminei o 2º grau, meu desejo era fazer o IME, fui parar na Turma 
  IME/ITA, abandonei as matérias tipo Biologia, História,...
  Entretanto, se este não for o seu objetivo, 
  não deixe de estudá-las. Principalmente, línguas e português. Alguns 
  colegas do meu tempo de IME estão na Microsoft (desenvolvendo 
  coisasfor Windows), porém eles continuaram estudando inglês e 
  espanhol. Pense bem, esta é uma decisão 
  difícil.
  Um abraço 
  Fábio Arruda
  
- Original Message - 
From: 
Gustavo 
Martins 
To: [EMAIL PROTECTED] 
Sent: Thursday, May 03, 2001 4:43 
PM
Subject: Aprendendo mat. sem perder 
o resto

Colegas:

Estou no 3º ano do Ens. Médio e percebi que se eu desejo aprender 
*bem* a Matemática e outras matérias exatas, tenho que ter dedicação 
quase exclusiva, ficando com pouquíssimo tempo para estudar os "outros" 
assuntos (biologia, geografia, etc). Porém, se eu tiver que fazer 
isso,posso me dar mal. Creio que alguns já passaram por esse 
problema e podem me dar algum tipo de sugestão para que eu possa 
aprender bem as matériasexatas e sobrar algum tempo para as 
outras. Qualquer ajuda serve.

Atenciosamente,
Gustavo


Re: raiz quadrada - novamente 2

2001-04-25 Por tôpico Alexandre F. Terezan



Acredito que a afirmação sqrt(4) = + 2 deve ser lida 
como:

sqrt(4) = 2 e sqrt(4) = -2, o que 
justificaria ser uma proposição FALSA, já que sqrt(4) = -2 é 
FALSO.

Isso porque, da mesma forma, quando dizemos que as raízes da 
equacao x^2- 4 = 0 são 2 e -2 (o que é VERDADEIRO), 
representamos como x = + 2.

Este "ou" de "mais ou menos" não representa o OU (v) da 
lógica matemática. 



  - Original Message - 
  From: 
  Fábio Arruda de Lima 
  To: [EMAIL PROTECTED] 
  Sent: Segunda-feira, 23 de Abril de 2001 
  23:27
  Subject: Re: raiz quadrada - novamente 
  2
  
  Olá amigos,
  Então vou aproveitar a dúvida do Nicks, para 
  tentar ajudar a outros. O Colégio São Bento no Rio, ensina lógica matemática 
  na 5ª série do 1º grau, matéria que eu fui estudar apenas na Turma IME/ETA 
  (Prof. Roquette - alguns desta lista devem lembrar dele) e, realmente, bem 
  estudado no IME na Curso de Engenharia de Computação. Entretanto, acho que há 
  um erro no Currículo Brasileiro de Matemática, por nãoser disciplina 
  obrigatória para todos os Colégios. Bom, feita a introdução, passemos ao que 
  interessa. Os primeiros conectivos lógicos importantes são o "E","OU" e 
  o "~" (lê-se Não). Em seguida, há que se 
  distinguir "Definições" e "Teoremas". Quando definimos algo,usamos o 
  conectivo lógico "se e somente se" (lê-se =). Por outro lado, quando 
  expressamos uma Teorema, normalmente, escrevemos a Hipótese "=" 
  Tese.Muito cuidado, no uso deste último. Implicações só caminham para 
  frente, nunca para trás. Há outros conectivos usados em computação como o "OU 
  EXCLUSIVO". 
  Como sugestão,seria interessanteler o 
  livro do Prof. Edgar de Alencar Filho - Lógica Matemática. Se eu não me 
  engano, o Gelson Iezzi tem alguns páginas no Livro I. Um abraço 
  galera.
  Fábio Arruda
  
  
  
  
  
  
- Original Message - 
From: 
Nicks 
To: [EMAIL PROTECTED] ; [EMAIL PROTECTED] 
Sent: Tuesday, April 24, 2001 4:45 
AM
Subject: Re: raiz quadrada - novamente 
2
Obrigado Fábio pelo alerta e, 
realmente atropelei a lógica matemática na forma que 
foi escrita; mas a minha dúvida é a veracidade ou 
não da proposição s , já que "p OU q" é verdadeiro quando 
pelo menos um deles é verdadeiro .[]'s 
NicksAt 11:33 23/4/2001 -0300, Fábio Arruda de Lima 
wrote:
Olá.Veja bem amigo Nicks, na 
  Lógica Matemática tem coisas que são distintas: "E", "OU", "ENTÃO" e 
  "Se e Somente Se".Para "E" ser verdade 
  é necessário que ambas as afirmativas sejam verdadeiras.Para "OU" ser verdade é necessário que uma delas seja 
  verdade ou as duas sejam. E veja os outros casos...Na sua tese estes conectivos lógicos se misturaram. Você 
  escreveu o seguinte:"s: 
  sqrt(4)= +2 ou seja s: sqrt(4) =2 ou sqrt(2)=-2 . Como sqrt(4) =2 é 
  verdadeira e sqrt(2)=-2 é falsa , concluímos que s é uma proposição 
  verdadeira ; ou seja sqrt(4)= +2 
  é verdadeiro logicamente "Obeserve o "OU" e o "E"; Veja se seu uso foi 
  realmente correto. Além disso, o termo "Ou seja" significa Tabela Verdade 
  do "então" ou do "se somente se"?! A verdadeira lógica matemática é 
  fundamentada na precisão do uso dos conectivos.Um abraço.Fábio
  - Original Message - 
From: Nicks 
To: [EMAIL PROTECTED] ; [EMAIL PROTECTED] 
Sent: Monday, April 23, 2001 9:28 PMSubject: Re: 
raiz quadrada - novamente 2Olá pessoal ,Agradeço ao 
Professor Jose Paulo Carneiro pelo seu esclarecimento,pois um colega de 
turma havia dito que sqrt(4)=+ 2 no campo dos complexos e, agora 
pelo que entendi esta afirmativa é falsa , já que 4 é um número real 
.Agora ,o que está me intrigando é o seguinte fato :considere a 
seguinte afirmativa :p: 98 ou 8=9 ; pelo estudo da Lógica 
Matemática temos que o valor lógico de p é verdadeiro .Agora 
considere a seguinte afirmativa s: sqrt(4)= +2 ou seja s: 
sqrt(4) =2 ou sqrt(2)=-2 . Como sqrt(4) =2 é verdadeira e sqrt(2)=-2 é 
falsa , concluímos que s é uma proposição verdadeira ; ou seja 
sqrt(4)= +2 é verdadeiro logicamente , ???!! .Diante de tal 
problemaeu e um colega discutimos e este amigo afirmou :" à luz 
da Lógica Matemática é uma afirmativa verdadeira , mas quando 
perguntamos sqrt(4) estamos interessado na raiz positiva ". Indaguei o 
seguinte :se s fosse sqrt(4) =2 ou sqrt(4)=5 , isto não seria verdadeiro 
? só devemos usar a Lógica Matemática quando alguém perguntar ? acredito 
que devemos usá-la sempre , ok ? . Foi exatamente aqui que este colega 
disse que no campo dos complexos sqrt(4) = +2 .O que 
vocês da lista acham desta discussão ? desculpem se este papo está se 
estendendo muito , mas isto não está me deixando 
sossegado.[]'s NicksAt 21:41 
17/4/2001 -0300, Jose Paulo Carneiro wrote:Muito 

Re: Algoritmo de equacao

2001-04-14 Por tôpico Alexandre F. Terezan



Vindo do senhor, sinto-me muito grato pelo 
elogio.

Aliás, a história da minha "descoberta" é muito 
simples.

Vem do seguinte (e fácil) probleminha:

 Sejam x1 e x2 as raízes da 
equacao x^2 + bx + c = 0. Sejam (x1)^3 e (x2)^3 as raízes da equacao x^2 + 
px + q = 0. Encontre p e q em funcao de b e c.

Como resposta temos: q = c^3; p = b^3 - 
3bc

Rearrumando a segunda equacao, vem: b^3 - 3bc - 
p = 0

Meu algoritmo nada mais é do que arrumar uma equacao de 
terceiro grau de forma que ela possa ser resolvida pela equacao de terceiro 
grau:

b^3 - 3bc - p = 0 , onde eu quero 
encontrar b, que corresponde ao oposto da soma das raízes cúbicas da 
equacao:

x^2 + px + q = 0 , onde q =c^3.

Espero ter sido claro...

[ ]'s

Alexandre Terezan


- Original Message - 

  From: 
  Jose Paulo 
  Carneiro 
  To: [EMAIL PROTECTED] 
  Sent: Sexta-feira, 13 de Abril de 2001 
  04:50
  Subject: Re: Algoritmo de equacao
  
  Quero acrescentar o seguinte:
  1) De qualquer forma, se o Alexandre redescobriu 
  sozinho o metodo de Cardano, ele estah de parabens!
  2) O Gugu, quando era aluno do ensino medio, 
  descobriu sozinho um metodo, diferente do de Cardano, para resolver "por meio 
  de radicais" as equacoes do terceiro e do quarto grau. Isto estah em algum 
  numero da Revista do Professor de Matematica.
  3) Para o 4o grau, o meu livro menciona apenas 
  superficialmente um metodo muito chato. O mais comumente mencionado eh o de 
  Ferrari, contemporaneo de Cardano. Gosto mais do metodo do Gugu.
  4) Fique claro que a resolucao por meio de 
  radicais tem um interesse mais teorico e historico. O mais pratico mesmo eh 
  usar metodos numericos que servem para qualquer grau (tambem estao no meu 
  livro, sem usar "Calculo").
  JP
  
  
  
  - Original Message - 
  
From: 
Luis 
Lopes 
To: [EMAIL PROTECTED] 
Sent: Thursday, April 12, 2001 4:35 
PM
Subject: Re: Algoritmo de equacao

Sauda,c~oes,

Somente duas observações:

1) as equações do 3o. grau não precisam ser incompletas. O 
algoritmo resolve equações do tipo
a_3x^3 + a_2x^2 + a_1x + a_0 = 0 . Só que o primeiro passo 
é transformá-la na equação ax^3 + bx + c = 0 
através de uma mudança de 
variáveis.

2) há também um algoritmo para resolver equações do 4o. 
grau. E um dos passos é resolver uma equação do 3o. grau.
O livro do JP deve falar disso.

[ ]'s
Lu'is

  -Mensagem Original- 
  De: 
  Jose Paulo 
  Carneiro 
  Para: [EMAIL PROTECTED] 
  Enviada em: Quinta-feira, 12 de Abril 
  de 2001 14:32
  Assunto: Re: Algoritmo de 
  equacao
  
  Isto eh o metodo conhecido como de Cardano 
  (embora a ideia original nao seja dele), e publicado no primeiro livro 
  impresso de Algebra, a Ars Magna de Cardano (1545). Veja o meu livro 
  Resolucao de Equacoes Algebricas, Ed. da Univ. Santa Ursula.
  So alguns detalhes:
  
  a) No passo 2, eh q, e nao -q.
  
  b) Nos passos 4 e 5, qundo voce diz 
  "encontramos raiz cubica", voce estah trabalhando nos reais ou complexos? 
  Se for nos complexos, seria necessario esclarecer qual das 3 raizes 
  cubicas se escolherah. Isto pode dar problema no passo 6, e voce achar 
  "raizes estranhas". Se voce so aceitar trabalhar nos reais, nao 
  conseguirah resolver equacoes simples, que so tem raizes 
  reais.
  O melhor ehsubstituir os passos 3 a 6 
  por:
  3) Encontre uma raiz y1 (real ou nao) da 
  equacao acima.
  4) Encontre uma raiz cubicaz de y1 
  (isto eh, qualquer complexo z tal que z^3=y1)
  5) Temos x1=z-p/(3z)
  
  c) Os passos 7 a 10 estao corretos (uma vez 
  achado x1), mas as outras raizes poderiam ser achadas diretamente. 
  Chamando u = - p/(3z), e w=cis(2pi/3), temos: x2= wz+w^2u; x3= w^2z+wu. 
  (Observe que w^2=1/w = conjugado de w)
  
  d) Observe tambem que x1=z+u; x2= wz+w^2u; 
  x3= w^2z+wu. Esta "danca" de 1, w, w^2 (que sao as raizes cubicas de 1) 
  foram responsaveis, historicamente, pela introducao do tema "permutacoes" 
  na resolucao de equacoes algebricas, e acabaram dando na Teoria de Galois 
  (1830), que explica quando uma equacao pode ser resolvida por uma 
  "formula", em termos das propriedades dos "grupos" de permutacoes das suas 
  raizes.
      
  JP
  
- Original Message - 
From: 
Alexandre F. Terezan 
To: OBM 

Sent: Thursday, April 12, 2001 
11:55 AM
Subject: Algoritmo de equacao

Por uma obra "do acaso", "descobri" (sem saber q já 
existia) um algoritmo que calcula as raízes de uma equacao do 
tipo:

ax^3 + bx + c = 0

Chamamos 

Re: Racionalizar

2001-03-29 Por tôpico Alexandre F. Terezan



seja k = raiz cúbica de 3

seja x^y = x elevado a y

Ora, do produto notável temos:a^3 - b^3 = (a - b)(a^2 + 
ab + b^2)

Substituindo a por k e b por 1, vem: k^3 - 1 = (k - 1)(k^2 + k 
+ 1)

Mas k^3 - 1 = 2 - 1 = 1

Entao, para racionalizarmos a fracao 1/(k - 1), basta 
multiplicarmos tanto o numerador quanto o numerador por (k^2 + k + 
1)...

Assim,teremos 1/(k - 1) = k^2 + k + 1

Espero ter ajudado.



  - Original Message - 
  From: 
  Davidson 
  Estanislau 
  To: obm 
  Sent: Quinta-feira, 29 de Março de 2001 
  13:43
  Subject: Racionalizar
  
  
   Caros colegas, como faço para 
  racionalizar a seguinte expressão:
  
   
  
  
   Obrigado.
  
   
Davidson


Re: fracoes

2001-03-22 Por tôpico Alexandre F. Terezan



Podemos generalizar todas as fracoes dadas para k 
/ [k + (n+2)], onde k é natural 18  k  92.

Ora, a fracao k / [k + (n+2)] é irredutível se nao há 
divisores comuns a {k} e {k + (n+2)}. 
Isso acontece necessariamente quando (n+2) é um primo que NAO 
divide k.
Logo, basta escolhermos um número (n+2) primo maior que 
91, pois este necessariamente nao dividirá nenhum k (visto que um número nao 
pode ser divisor de outro número menor do que ele).

(n + 2) = 97 -- n = 95 é uma solucao 
possível.

Espero ter 
ajudado...

  - Original Message - 
  From: 
  josimat 
  To: OBM 
  Sent: Sexta-feira, 23 de Março de 2001 
  00:26
  Subject: fracoes
  
  Ola 
  pessoal!
  Dois amigos meus querem 
  comprar o livro "Problemas Selecionados de Matematica" do Raul Agostinho e do 
  Antônio Luis, alguem sabe como?
  Esses mesmos amigos, 
  passaram-me um problema que nao consegui resolver. Alguem pode 
  ajudar?
  19/n+21 , 20/n+22 , 
  21/n+23 , ... , 91/n+93 (com 73 fracoes)
  qual o valor de n para 
  que todas essas fracoes sejam irredutiveis?
  []s, 
Josimar


EQUILÁTERO

2001-03-17 Por tôpico Alexandre F. Terezan



Provar (com noçoes de 2o grau de preferência) que, dada 
uma circunferencia de raio R, o triangulo nela inscrito de maior área é o 
triângulo equilátero de ladoigual a R x (sqrt 3)

Obs: x = produto
 sqrt 3 = raiz 
quadrada de 3 


Re: 260

2000-12-07 Por tôpico Alexandre F. Terezan




Bn = B(n-1) x (n-1) + B(n-2) x (n-2)

Seja An todas as arrumacoes de n possíveis (pela regra), ou seja,
n {An} = Bn

* A primeira parcela [B(n-1) x (n-1)] se refere às (n-1) posicoes em q 
podemos colocar o enésimo termo em cada uma das arrumaçoes de A(n-1), fazendo 
valer a regra.

* A segunda parcela é um pouco mais complexa. 

Ela se refere aos casos particulares em que com os primeiros (n-1) termos 
temos APENAS UM par de algarismos desobedecendo a regra, ou seja, temos 2 
algarismos consecutivos em ordem.

Assim, podemos colocar o último algarismo entre esses dois, fazendo a regra 
voltar a valer.

Um dos pares de números consecutivos de 1 a (n-1) pode ser considerado como 
um algarismo apenas, fazendo valer a regra para A(n-2), o q nos dará arrumacoes 
onde haverá apenas UM par desobedecendo a regra (o par q escolhemos). Nesse 
caso, há B(n-2) maneiras possíveis.

Ora, podemos fazer valer a regra, desta maneira, com qualquer par de 
números consecutivos (em ordem) de 1 a (n-1). Como existem (n-2) pares neste 
conjunto, e há B(n-2) maneiras para cada par, prova-se a segunda parcela.

Assim, Bn = B(n-1) x (n-1) + B(n-2) x (n-2)
Onde vc ficou surpreso, Nicolau?

- Original Message - 
From: "Nicolau C. Saldanha" [EMAIL PROTECTED]
To: [EMAIL PROTECTED]
Sent: Quinta-feira, 7 de Dezembro de 2000 00:35
Subject: 260
Não, não é o número de pontos de ninguém.É o número de 
membros da nossa lista: 260.Eu verifico este número periodicamente e esta é 
a 1a vezque observo um número 250.Mas mudando de 
assunto...Arrumamos em fila n bolinhas numeradas de 1 a n.De quantas 
formas podemos fazê-lo sem que:1 fique imediatamente antes de 2,2 fique 
imediatamente antes de 
3, ...(n-1) fique 
imediatamente antes de n?Chamemos a resposta de BnEstas são as 
únicas restrições. Não é proibido que 2 venha logo antes de 
1.TemosB2 = 1 (21)B3 = 3 (132, 213, 321)B4 = 11 (1324, 
1432, 2143, 2413, 2431, 3142, 3214, 3241, 4132, 4213, 4321)O problema 
não é tão difícil, mas há algo que me surpreendeu na resposta.[]s, 
N.


Re: Combinatória

2000-11-25 Por tôpico Alexandre F. Terezan




A minha resposta está errada, Morgado.

Desculpe pela desatencao...

O erro está nessa passagem:

 Nessa situacao, há 7!/(3! x 2!) = 420 modos de dispormos os 
algarismos (anagramas de "44488XY").  Além disso, há 8 
possibilidades para X (X diferente de 4 e 8) e 7 possibilidades para Y 
(Y diferente de X, 4 e 8).  Logo, temos 420 x 7 x 8 = 23520 
possibilidades.

No momento que eu assumo "anagramas" de 44488XY, as possibilidades de 
escolha para X e Y se reduzem para (7 x 8)/2.

A divisao por 2 que eu tinha omitido é necessária, visto que, do contrário, 
estaremos considerando duas vezes todos os "anagramas" mencionados.

Exemplo: Seja o anagrama 444X88Y. Ora, para X = 1, Y = 2, temos o número 
4441882. 
 Seja agora o anagrama 444Y88X, para X = 2, Y = 1, dando 
o número 4441882.

Ou seja, temos 2 anagramas "diferentes" que dao o mesmo número. 

De modo geral, há sempre23520 pares de anagramas 
onde as posicoes de X e Y se invertem. 

Esses anagramas "invertidos" irao sempre duplicar o número real de números 
possíveis, já q para cada anagrama invertido, temos um par de opcoes (X = a, Y = 
b) (X = b, Y = a) q irao dar o mesmo número.

Dessa forma, há 11760 anagramas 44488XY nas condicoes mencionadas. 
Excetuando os 420 q começam com zero, temos 11340 modos de 
escolhermos algarismos segundo o enunciado quando X diferente de Y.

Para X = Y valem os 1620 q encontrei na solucao original.

Assim, há realmente 12960 maneiras.

Obrigado pela válida observacao, professor.

[]'s, Alexandre Terezan


- Original Message - 
From: "Augusto Morgado" [EMAIL PROTECTED]
To: [EMAIL PROTECTED]
Sent: Sábado, 25 de Novembro de 2000 09:19
Subject: Re: Combinatória
Outro modo de fazer seria: Dos 7 lugares, devemos escolher tres 
paracolocar os quatros, o que pode ser feito de C7,3=35 modos. Dos 
quatrolugares que sobraram, devemos escolher dois para botar os oitos, o 
quepode ser feito de C4,2=6 modos. Agora temos duas casas a preencher, 
oque pode ser feito de 8x8=64 modos e a resposta seria 
35x6x64=13440modos. Devemos descontar os começados em 0,que saoC6,3 x 
C3,2 x 8 = 20x3x8=480 e a resposta eh 12960.Como as respostas deram 
diferentes, uma estah certa e a autra nao. Qualdelas e onde estah o 
erro?"Alexandre F. Terezan" wrote:  Imagine o número 
44488XY de 7 dígitos, onde X é um algarismo diferente de 4 e 8. 
 1o caso: X diferente de Y  Nessa situacao, há 7!/(3! x 
2!) = 420 modos de dispormos os algarismos (anagramas de 
"44488XY").  Além disso, há 8 possibilidades para X (X diferente 
de 4 e 8) e 7 possibilidades para Y (Y diferente de X, 4 e 8). 
 Logo, temos 420 x 7 x 8 = 23520 possibilidades.  
Deve-se desconsiderar os casos em q o primeiro algarismo é zero. Existem 
6!/(3! x 2!) modos de arrumarmos "44488A", onde A diferente de 0, 4 e 8: 
60 x 7 = 420 maneiras onde 0 é o primeiro algarismo.  
Assim, há 23100 maneiras de dispormos 44488XY.  2o caso: X = 
Y  Aqui, temos 7!/(3! x 2! x 2!) = 210 maneiras de dispormos 
"44488XX". Como X diferente de 4 e 8, há 8 "X" possíveis, nos dando 1680 
casos.  Desses 1680, tiremos os casos onde o primeiro algarismo 
é zero. Neste caso há 6!/(3! x 2!) possibilidades de arrumarmos "444880" 
a partir do primeiro zero, o q nos dá 60 casos impossíveis. 
 Logo, 1620 casos satisfazem, quando X = Y.  TOTAL: 
23100 + 1620 = 24720 possibilidades.  - Original Message 
- From: "ricardopanama" [EMAIL PROTECTED] 
To: [EMAIL PROTECTED] Sent: 
Sexta-feira, 24 de Novembro de 2000 17:58 Subject: Combinatória 
 Agrdeço a quem responder este problema de combinatória: 
 Quantos são os algarismos de 7 dígitos nos quais o algarismo 4 
figura exatamente 3 vezes e o algarismo 8 exatamente 2 vezes? 
 Abrços.  
__ 
Preocupado com vírus? Crie seu e-mail grátis do BOL com antivírus ! http://www.bol.com.br


Re: Combinatória

2000-11-24 Por tôpico Alexandre F. Terezan

Imagine o número 44488XY de 7 dígitos, onde X é um algarismo diferente de 4
e 8.

1o caso: X diferente de Y

Nessa situacao, há 7!/(3! x 2!) = 420 modos de dispormos os algarismos
(anagramas de "44488XY").

Além disso, há 8 possibilidades para X (X diferente de 4 e 8) e 7
possibilidades para Y (Y diferente de X, 4 e 8).

Logo, temos 420 x 7 x 8 = 23520 possibilidades.

Deve-se desconsiderar os casos em q o primeiro algarismo é zero. Existem
6!/(3! x 2!) modos de arrumarmos "44488A", onde A diferente de 0, 4 e 8: 60
x 7 = 420 maneiras onde 0 é o primeiro algarismo.

Assim, há 23100 maneiras de dispormos 44488XY.

2o caso: X = Y

Aqui, temos 7!/(3! x 2! x 2!) = 210 maneiras de dispormos "44488XX". Como X
diferente de 4 e 8, há 8 "X" possíveis, nos dando 1680 casos.

Desses 1680, tiremos os casos onde o primeiro algarismo é zero. Neste caso
há 6!/(3! x 2!) possibilidades de arrumarmos "444880" a partir do primeiro
zero, o q nos dá 60 casos impossíveis.

Logo, 1620 casos satisfazem, quando X = Y.


TOTAL: 23100 + 1620 = 24720 possibilidades.





- Original Message -
From: "ricardopanama" [EMAIL PROTECTED]
To: [EMAIL PROTECTED]
Sent: Sexta-feira, 24 de Novembro de 2000 17:58
Subject: Combinatória


Agrdeço a quem responder este problema de combinatória:


Quantos são os algarismos de 7 dígitos nos quais o
algarismo 4 figura exatamente 3 vezes e o algarismo 8
exatamente 2 vezes?


Abrços.


__
Preocupado com vírus? Crie seu e-mail grátis do BOL com antivírus !
http://www.bol.com.br







probabilidade!!

2000-11-20 Por tôpico Alexandre F. Terezan




Um fazendeiro convida para a sua casa um amigo seu, através de um 
telefonema. As duas únicas informações sobre o caminho são:

  
i. Ao longo do percurso, existem 10 "trifurcações" da estrada (3 
subdivisões a partir de 1 pré-existente), sendo que para chegar-se à casa 
deve-se escolher a subdivisão apropriada a cada trifurcação;

  
  
ii . Em 2 trifurcações deve-se virar à esquerda, em 3 deve-se virar à 
direita e em 5 deve-se seguir em frente (subdivisão central). 

Obviamente, é pouco provável que o amigo do fazendeiro consiga chegar à 
casa dele com estas informações apenas. Chame a probabilidade de sucesso do 
amigo nestas condições de P. Por outro lado, seria ainda mais difícil se a 
segunda informação (ii) fosse omitida. Chame a probabilidade de sucesso do amigo 
nestas condições de P. Encontre a razão P/P.


probabilidade

2000-11-19 Por tôpico Alexandre F. Terezan



Um fazendeiro convida para a sua casa um amigo seu, através de um 
telefonema. As duas únicas informações sobre o caminho são:

  
i. Ao longo do percurso, existem 10 "trifurcações" da estrada (3 
subdivisões a partir de 1 pré-existente), sendo que para chegar-se à casa 
deve-se escolher a subdivisão apropriada a cada trifurcação;

  
  
ii . Em 2 trifurcações deve-se virar à esquerda, em 3 deve-se virar à 
direita e em 5 deve-se seguir em frente (subdivisão central). 

Obviamente, é pouco provável que o amigo do fazendeiro consiga chegar à 
casa dele com estas informações apenas. Chame a probabilidade de sucesso do 
amigo nestas condições de P. Por outro lado, seria ainda mais difícil se a 
segunda informação (ii) fosse omitida. Chame a probabilidade de sucesso do amigo 
nestas condições de P. Encontre a razão P/P.


Re: Duas questõezinhas!!!

2000-11-15 Por tôpico Alexandre F. Terezan




Resolvendo as questoes 1 e 2 ,ou tentando :-) ...

1) Sejam:

a = percentagem de entrevistados que consomem A, mas nao consomem nem B nem 
C
b = percentagem de entrevistados que consomem B, mas nao consomem 
nemA nem C

c= percentagem de entrevistados que consomem C, mas nao consomem 
nemA nem B
m = percentagem de entrevistados que consomem A e B, mas nao consomem 
C

n= percentagem de entrevistados que consomemB e C, mas nao 
consomem A
p = percentagem de entrevistados que consomemA e C, mas nao consomem 
B
x = percentagem de entrevistados que consomemA, B e C

A percentagem de entrevistados que consomem algo é 85%, logo:

a + b + c + m + n + p + x = 85 (I)

Por outro lado, se tomarmos que 32% não consomem A, 44% não consomem B e 
34% não consomem C, temos:

b + c + n + 15 = 32 (32% não consomem A)
a + b + m + 15 = 34 (34% não consomem C)
a + c + p + 15 = 44 (44% não consomem B)

Somando as três equacoes, temos:

2a + 2b + 2c + m + n + p = 65 (II)

Fazendo (I - II), vem:

x - (a + b + c) = 20, ou:

x = 20 + (a+b+c)

x será mínimo quando (a+b+c) = 0, ou seja, em tal caso, x = 20%

Basta verificar se é possível que haja a = b = c = (a+b+c) = 0

Isso é possível, resultando em m = 29%, n = 27% e p = 39%

Logo, a porcentagem mínima de entrevistados que consomem A, B e C é 
de 20%.

2) Resposta: d.

 a) Pelo princípio de Dirichlet, vemos qessa 
afirmaçao é verdadeira(15  1 x 12) .

 b)Pelo princípio de Dirichlet, vemos qessa 
afirmaçao também é verdadeira ( 15  2 x 7).

 e) Sejam as pessoas (A,B,C,D,E,F,G,H,I,J,K,L,M,N,O) 
Imagine que a idade da pessoaA seja igual a 14a + a',a idade da 
pessoa B seja igual a 14b + b' e assim sucessivamente, até que a idade da pessoa 
O seja igual a 14o + o'.Todo Y' deve obedecer0 = y' = 13 


Ora, há 14 y' possíveis e 15 pessoas. Logo, há pelo menos duas pessoas q 
possuam o mesmo y'.

Sem perda de generalidade, seja a' = b'.

A diferenca de idade de A e B será: 14a + a' - 14b - b' =14a - 14b + a' - 
a' = 14 (a - b) , q é um múltiplo de 14.

Logo, esta afirmativa também é verdadeira.

 c) Sejay o número de pessoas q a pessoaY 
conhece, e sejam as pessoas (A,B,C,D,E,F,G,H,I,J,K,L,M,N,O).

Sem perda de generalidade, seja a= 14, pelo enunciado.

Ora, nenhuma das outras pessoas pode conhecer ZERO pessoas, visto q A 
conhece todas (reciprocidade mencionada).

Caso i) Alguma das outras pessoas (q nao A) conhece 14 pessoas. Isto 
tornaa afirmativa c verdadeira.

Caso ii) Nenhuma das outras pessoas (q nao A) conhece 14 pessoas. Assim, há 
13 "y" possíveis (1 = y = 13) para 14 pessoas, o q faz com q pelo menos 
2 pessoas conhecam o mesmo número de pessoas.

Logo, a afirmativa c é verdadeira.

 
 d) Sejay o número de pessoas q a 
pessoaY conhece, e sejam as pessoas (A,B,C,D,E,F,G,H,I,J,K,L,M,N,O).

Sem perda de generalidade, sejaa = 0 (pelo enunciado)

Neste caso, visto q A nao conhecenenhuma pessoa, nenhuma das outras 
pessoas pode conhecer 14 pessoas (reciprocidade mencionada).

Caso i) Alguma das outras pessoas (q nao A) conheceZERO pessoas. Isto 
tornaa afirmativad falsa.

Caso ii)Cada uma das outras pessoas (q nao A) conhece mais de uma 
pessoa. 

Assim, há 13 "y"s possíveis (1= y = 13) para 14 pessoas, o q faz 
com q pelo menos 2 pessoas conhecam o mesmo número de pessoas.

Isto torna a afirmativa dFALSA. 

UFA, espero ter ajudado.

[]'s , Alexandre Terezan.

- Original Message - 
From: "Augusto Morgado" [EMAIL PROTECTED]
To: [EMAIL PROTECTED]
Sent: Quarta-feira, 15 de Novembro de 2000 09:12
Subject: Re: Duas questõezinhas!!!
Bonitas questoes. A primeira (com outros numeros) caiu num 
vestibular daUERJ e nao houve 50 candidatos que conseguissem 
resolve-la.Via Lux wrote:  Olá pessoal,  
Aí vão duas questões que deixam muitos alunos confusos...  E vcs 
que dizem?  1) Numa pesquisa sobre o consumo dos produtos A, B e 
C, obteve-se o seguinte resultado: 68% dos entrevistados consomem A, 56% 
consomem B, 66% consomem C e 15% não consomem nenhum dos produtos. Qual 
a porcentagem mínima de entrevistados que consomem A,B e C? 
 2) Considerando que em uma festa há 15 pessoas, não podemos afirmar 
que:  a)pelo menos duas nasceram no mesmo mês do ano. 
 b)pelo menos três nasceram no mesmo dia da semana  c)se 
uma das pessoas conhece as demais então existem pelo menos duas com o 
mesmo número de conhecidos (o conhecer alguém é recíproco)  d)se 
uma pessoa não conhece ninguém então pode não existirem duas pessoas com 
o mesmo número de conhecidos (o conhecer alguém é recíproco)  e) 
a diferença de idade "em anos" de duas delasé um múltiplo de 14 
 
Lembranças a todos, 
 
Fui!  Luciano M. Filho


Re: combinatória

2000-10-29 Por tôpico Alexandre F. Terezan

A diferença entre Z* e Z*+ reside no fato de q o primeiro conjunto contém os
inteiros menores do que ZERO enquanto o segundo nao...

O meu problema nao é exatamente esse...

O fato é q a definicao mais comum (nao necessariamente a correta, isso é o q
eu quero saber) para número positivo q eu conheço seria todo número MAIOR do
q zero (o q, obviamente, exclui o próprio zero)

- Original Message -
From: "Eduardo Favarão Botelho" [EMAIL PROTECTED]
To: [EMAIL PROTECTED]
Sent: Domingo, 29 de Outubro de 2000 21:53
Subject: combinatória



O ZERO tb faz parte dos inteiros não negativos, é verdade. Ele é elemento de
ambos os conjuntos: o dos positivos e o dos negativos. Caso contrário, não
existiria diferença entre os conjuntos Z* (inteiros não nulos) e Z*+
(inteiros positivos não nulos), por exemplo.

Abraços, Eduardo

-Mensagem original-
De: Alexandre F. Terezan [EMAIL PROTECTED]
Para: [EMAIL PROTECTED] [EMAIL PROTECTED]
Data: Domingo, 29 de Outubro de 2000 12:46
Assunto: Re: combinatória


Essa sempre foi uma dúvida que eu tive...

Pra mim, o ZERO participaria do conjunto dos inteiros "Não-negativos" mas
nao do conjunto dos inteiros "positivos"...

O q está certo e o q está errado?






Re: probabilidade

2000-10-24 Por tôpico Alexandre F. Terezan



As possibilidades de sexo para qualquer casal, no caso de 2 
filhos, sao:

HH, HM, MH e MM, onde a primeira letra representa o filho mais 
velho...
H = homem
M = mulher

No caso do casal A, como pelo menos um filho é homem, restam 
as seguintes possibilidades: HH, HM e MH
Como apenas 1 interessa das tres possiveis, a = 
1/3

No caso do casal B, onde o filho mais velho é homem, restam as 
seguintes possibilidades: HH e HM
Como apenas 1 interessa das duas possiveis,b = 
1/2

Logo, 1/3 = a  b = 1/2

  - Original Message - 
  From: 
  Filho 
  
  To: discussão de 
  problemas 
  Sent: Segunda-feira, 23 de Outubro de 
  2000 15:37
  Subject: probabilidade
  
  Os casais A e B têm dois filhos cada um. 
  Sabe-se que o casal A tem um filho homem e que o filho mais velho do casal B 
  também é homem. Se a e b indicam, respectivamente, as probabilidades de que os 
  dois filhos do casal A sejam homens e que os dois filhos do casal B também 
  sejam homens, então:
  a) a  
  b 
  b) a = 
  b 
  c) a  
  b 
  d) a + b = 
  1 
  e) n.r.a


Re: saída

2000-10-08 Por tôpico Alexandre F. Terezan

Só gostaria de fazer uma observaçao:

A probabilidade Z é na verdade 27,1 %

Isso pq vc se esqueceu do caso em q ele ganha em 2 sorteios e perde em um
deles...

[ ]'s, Alexandre Terezan.

- Original Message -
From: "Douglas C. Andrade" [EMAIL PROTECTED]
To: [EMAIL PROTECTED]
Sent: Sábado, 22 de Junho de 1996 22:21
Subject: Re: saída


Deixe-me ver se entendi bem:

A probabilidade X vale 3/10 (tres chances em dez) por razoes obvias, ou
X=30%

Na segunda opcao, a chance do apostador ganhar os dois sorteios e
1/10*2/10=2/100 (a*b=a vezes b)
A chance de ele perder o sorteio para o qual comprou somente um bilhete e
ganhar o outro vale 9/10*2/10=18/100.
A chance de ele perder o sorteio para o qual comprou dois bilhetes e ganhar
o outro vale 8/10*1/10=8/100.

Entao a chance de ele ganhar alguma coisa no segundo caso vale 28/100, ou
Y=28%.

No terceiro caso, a chance de o apostador ganhar todos os tres sorteios vale
1/10*1/10*1/10=1/1000
A chance de ele ganhar um dos sorteios (tres opcoes) e perder os outros dois
vale 3*1/10*9/10*9/10=243/1000.
Entao sua chance de ganhar algo no terceiro caso vale 244/100, ou Z=24,4%

Espero ter ajudado

Douglas
-Mensagem original-
De: Filho [EMAIL PROTECTED]
Para: discussão de problemas [EMAIL PROTECTED]
Data: Domingo, 8 de Outubro de 2000 17:05
Assunto: saída


Um  apostador tem três opções para participar de certa modalidade de
jogo, que consiste no
sorteio aleatório de um número dentre dez.
-primeira opção: comprar três números para um único sorteio.
-segunda opção: comprar dois números para um sorteio e um número para um
segundo sorteio.
-terceira opção: comprar um número para cada sorteio, num total de três
sorteios.

Se X,Y,Z representam as probabilidades de o apostador ganhar algum
prêmio, escolhendo, respectivamente, a primeira, a segunda ou a terceira
opções, qual a relação entre X,Y e Z.





Re: comentários

2000-09-30 Por tôpico Alexandre F. Terezan

Olá,

De acordo com a nova situacao proposta pelo Nicolau:

Chamando de V1 a face vermelha do cartao bicolor e de V2 e V3 as faces
vermelhas do cartao todo vermelho.

Se a face vista pelo juiz é vermelha, assume-se q há igual probabilidade de
que a face vista por ele seja V1, V2 ou V3 (1/3 de probabilidade para cada).

Dessa forma, se a face vista pelo juiz for V1  (1/3 de chances), entao o
jogador verá uma face amarela.

Se a face vista pelo juiz for V2 ou V3 (2/3 de probabilidade), entao o
jogador verá uma face vermelha.

Assim sendo, a probabilidade de que o jogador veja uma face amarela é de 1/3
apenas, contra 2/3 de probabilidade de que a face vista por ele seja
vermelha.

[ ]'s, Alexandre Terezan

- Original Message -
From: "Nicolau C. Saldanha" [EMAIL PROTECTED]
To: [EMAIL PROTECTED]
Sent: Sábado, 30 de Setembro de 2000 08:04
Subject: Re: comentários




On Sat, 30 Sep 2000, Alexandre F. Terezan wrote:

 Olá,

 Aparentemente a resposta é simples.  Para q o enunciado ocorra,
primeiramente
 o juiz deverá escolher o cartao bicolor (probabilidade de 1/3) e, além
disso,
 este cartao deverá ter a sua cor vermelha voltada para o juiz (1/2 de
 probabilidade)

 Assim, a probabilidade geral é de 1/2 * 1/3 = 1/6.


   Um juiz de futebol possui três cartões no bolso. Um é todo amarelo,
outro é
   todo vermelho e o terceiro é vermelho de um lado e amarelo do outro. Num
   determinado lance, o juiz retira, ao acaso, um cartão do bolso e mostra
a
   um jogador. Determine a probabilidade da face que o juiz vê ser vermelha
e
   da outra face mostrada ao jogador ser amarela.


O Alexandre tem razão, claro.  Uma variante mais interessante seria igual,
exceto pela última frase, que fica assim:

Determine a probabilidade de que a face mostrada ao jogador seja amarela
dado que a face que o juiz vê é vermelha.






Re: comentários

2000-09-29 Por tôpico Alexandre F. Terezan



Olá,

Aparentemente a resposta é simples. 
Para q o enunciado ocorra, primeiramente o juiz deverá 
escolher o cartao bicolor (probabilidade de 1/3) e, além disso, este cartao 
deverá ter a sua cor vermelha voltada para o juiz (1/2 de 
probabilidade)

Assim, a probabilidade geral é de 1/2 * 1/3 = 
1/6.

[ ]'s, Alexandre Terezan.

  - Original Message - 
  From: 
  Filho 
  
  To: discussão de 
  problemas 
  Sent: Sexta-feira, 29 de Setembro de 2000 
  22:59
  Subject: comentários
  
  Um juiz de futebol possui três cartões no 
  bolso. Um é todo amarelo, outro é todo vermelho e o terceiro
  é vermelho de um lado e amarelo do outro. Num 
  determinado lance, o juiz retira, ao acaso, um cartão do bolso e mostra a um 
  jogador. Determine a probabilidade da face que o juiz vê ser vermelha e da 
  outra face mostrada ao jogador ser 
  amarela.


Re: Problema

2000-08-19 Por tôpico Alexandre F. Terezan



Encontrei uma resposta genérica pra esse problema (q aliás foi 
proposto há muito tempo na lista) mas vou enunciar o caso particular abordado.
A resposta genérica é de fácil deduçao.

Utilizarei a#n como notaçao para a índice n, uma vez que * 
representará multiplicacoes.

a#n = 6^n + 8^n

a#93/a#49 = (6^93 + 4^93)/(6^49 + 4^49) = 
= [2^93 (3^93 + 4^93)] / [2^49 (3^49 + 4^49)] =
=2^44 [(3^93 + 4^93)/(3^49 + 
4^49)] 
(1) 


Seja a#93/a#49 = a#49 *k + R, ondek é natural e R é o resto 
procurado.

Assim 2^44 [(3^93 + 4^93)/(3^49 + 4^49)] = 2^49 * (3^49 + 4^49)k + 
R (2)

Seja 3^49 + 4^49 =a e 
3^93 + 4^93 = b.

Dessa forma, R = 2^44 * (b/a) - 2^49 *a * k

Ou, R = 2^44 * [ (b/a) - 32ak ] -- R = 2^44 * [ (b - 32k * a^2) / 
a] 

Ora,a é um natural ímpar. Portanto,a nao divide 
2^44.

Portnato, para que R seja natural,
o naturala necessariamente divide (b - 32k * a^2).

Como b é ímpar e (32k * a^2) é par, (b - 32k * a^2) é ímpar.

Numa divisao exata, se dividendo e divisor sao ambos ímpares, entao o 
quociente também será ímpar.

Assim sendo, deve haver um natural ímpar p, tal que R = 2^44 * p

Substituindo R por 2^44 * p em (2), temos:

2^44 * (b/a) = 2^49 * ak + 2^44 * p

Daí, 2^44 * [ (b/a) - p ] = 2^49 * ak

Ou, [ 2^44 * (b - ap) ] / a = 2^49 * ak

Como 2^29 * ak é um inteiro postivo, a deve dividir [ 2^44 * (b - 
ap) ].

2^44 nao é divisível por a, uma vez que a é um natural ímpar.

Daí, conclui-se que a deve dividir (b - ap).

Para isso, é necessario e suficiente que a divida 
b. (CONCLUSAO 1)

Mas, de (1), a#93/a#49 = 2^44 * [ (3^93 + 
4^93) / (3^49 + 4^49) ] = 
= 2^44 * (b/a)

Como b é divisível por a (conclusao 1), entao [ 2^44 * (b/a)] é um inteiro 
positivo, o que nos mostra que a#93 é divisível por a#49.

Assim sendo, o resto de a#93 / a#49 é ZERO.

Resposta: ZERO.

Espero ter ajudado, apesar da demora.

Saudaçoes Tricolores, Alexandre Terezan.

- Original Message - 
From: "Eduardo Quintas da Silva" [EMAIL PROTECTED]
To: [EMAIL PROTECTED]
Sent: Sexta-feira, 4 de Agosto de 2000 22:21
Subject: Problema
Seja a*n = 6^n + 8^n. Calcule o resto de a*93 / 
a*49.Entende-se por a*n : a índice 
n.


Re: Problema

2000-08-19 Por tôpico Alexandre F. Terezan

Desculpem-me pela asneira...

Alguém conseguiu resolver tal problema?

- Original Message -
From: "Ecass Dodebel" [EMAIL PROTECTED]
To: [EMAIL PROTECTED]
Sent: Sábado, 19 de Agosto de 2000 18:21
Subject: Re: Problema





From: "Alexandre F. Terezan" [EMAIL PROTECTED]
Reply-To: [EMAIL PROTECTED]
To: "OBM" [EMAIL PROTECTED]
Subject: Re: Problema
Date: Sat, 19 Aug 2000 15:16:27 -0300

Encontrei uma resposta genérica pra esse problema (q aliás foi proposto há
muito tempo na lista) mas vou enunciar o caso particular abordado.
  A resposta genérica é de fácil deduçao.

Utilizarei a#n como notaçao para a índice n, uma vez que * representará
multiplicacoes.

a#n = 6^n + 8^n

a#93/a#49 = (6^93 + 4^93)/(6^49 + 4^49) =
= [2^93 (3^93 + 4^93)] / [2^49 (3^49 + 4^49)] =
= 2^44 [(3^93 + 4^93)/(3^49 + 4^49)]  (1)

Seja a#93/a#49 = a#49 * k + R, onde k é natural e R é o resto procurado.

Olá Alexandre,

perceba que a#49 é inteiro, e portanto a#49*k somado a R é inteiro, ou seja,
você está supondo que a#93/a#49 é inteiro. Um jeito de fazer a divisão
euclidiana é a#93 = a#49*k + R, com Ra#49

Obrigado!




Assim 2^44 [(3^93 + 4^93)/(3^49 + 4^49)] = 2^49 * (3^49 + 4^49)k + R  (2)

Seja 3^49 + 4^49 = a   e   3^93 + 4^93 = b.

Dessa forma, R = 2^44 * (b/a) - 2^49 * a * k

Ou, R = 2^44 * [ (b/a) - 32ak ] --  R = 2^44 * [ (b - 32k * a^2) / a]

Ora, a é um natural ímpar. Portanto, a nao divide 2^44.

Portnato, para que R seja natural,
o natural a necessariamente divide (b - 32k * a^2).

Como b é ímpar e (32k * a^2) é par, (b - 32k * a^2) é ímpar.

Numa divisao exata, se dividendo e divisor sao ambos ímpares, entao o
quociente também será ímpar.

Assim sendo, deve haver um natural ímpar p, tal que R = 2^44 * p

Substituindo R por 2^44 * p em (2), temos:

2^44 * (b/a) = 2^49 * ak + 2^44 * p

Daí,  2^44 * [ (b/a) - p ] = 2^49 * ak

Ou, [ 2^44 * (b - ap) ] / a = 2^49 * ak

Como 2^29 * ak é um inteiro postivo, a deve dividir [ 2^44 * (b - ap) ].

2^44 nao é divisível por a, uma vez que a é um natural ímpar.

Daí, conclui-se que a deve dividir (b - ap).

Para isso, é necessario e suficiente que a divida b. (CONCLUSAO 1)

Mas, de (1), a#93/a#49 = 2^44 * [ (3^93 + 4^93) / (3^49 + 4^49) ] =
= 2^44 * (b/a)

Como b é divisível por a (conclusao 1), entao [ 2^44 * (b/a)] é um inteiro
positivo, o que nos mostra que a#93 é divisível por a#49.

Assim sendo, o resto de a#93 / a#49 é ZERO.

Resposta: ZERO.

Espero ter ajudado, apesar da demora.

Saudaçoes Tricolores, Alexandre Terezan.

- Original Message -
From: "Eduardo Quintas da Silva" [EMAIL PROTECTED]
To: [EMAIL PROTECTED]
Sent: Sexta-feira, 4 de Agosto de 2000 22:21
Subject: Problema


Seja a*n = 6^n + 8^n. Calcule o resto de a*93 / a*49.

Entende-se por a*n : a índice n.




Get Your Private, Free E-mail from MSN Hotmail at http://www.hotmail.com





Re: Problema

2000-08-19 Por tôpico Alexandre F. Terezan

DE FATO a#93/a#49 NÃO é inteiro, como se vê abaixo.

(3^93 + 4^93) $ 3 (mod 7)
(3^49 + 4^49) $ 4 (mod 7)

$ representa congruência

Novamente desculpem-me pela asneira anterior.

- Original Message -
From: "Ecass Dodebel" [EMAIL PROTECTED]
To: [EMAIL PROTECTED]
Sent: Sábado, 19 de Agosto de 2000 18:21
Subject: Re: Problema





From: "Alexandre F. Terezan" [EMAIL PROTECTED]
Reply-To: [EMAIL PROTECTED]
To: "OBM" [EMAIL PROTECTED]
Subject: Re: Problema
Date: Sat, 19 Aug 2000 15:16:27 -0300

Encontrei uma resposta genérica pra esse problema (q aliás foi proposto há
muito tempo na lista) mas vou enunciar o caso particular abordado.
  A resposta genérica é de fácil deduçao.

Utilizarei a#n como notaçao para a índice n, uma vez que * representará
multiplicacoes.

a#n = 6^n + 8^n

a#93/a#49 = (6^93 + 4^93)/(6^49 + 4^49) =
= [2^93 (3^93 + 4^93)] / [2^49 (3^49 + 4^49)] =
= 2^44 [(3^93 + 4^93)/(3^49 + 4^49)]  (1)

Seja a#93/a#49 = a#49 * k + R, onde k é natural e R é o resto procurado.

Olá Alexandre,

perceba que a#49 é inteiro, e portanto a#49*k somado a R é inteiro, ou seja,
você está supondo que a#93/a#49 é inteiro. Um jeito de fazer a divisão
euclidiana é a#93 = a#49*k + R, com Ra#49

Obrigado!




Assim 2^44 [(3^93 + 4^93)/(3^49 + 4^49)] = 2^49 * (3^49 + 4^49)k + R  (2)

Seja 3^49 + 4^49 = a   e   3^93 + 4^93 = b.

Dessa forma, R = 2^44 * (b/a) - 2^49 * a * k

Ou, R = 2^44 * [ (b/a) - 32ak ] --  R = 2^44 * [ (b - 32k * a^2) / a]

Ora, a é um natural ímpar. Portanto, a nao divide 2^44.

Portnato, para que R seja natural,
o natural a necessariamente divide (b - 32k * a^2).

Como b é ímpar e (32k * a^2) é par, (b - 32k * a^2) é ímpar.

Numa divisao exata, se dividendo e divisor sao ambos ímpares, entao o
quociente também será ímpar.

Assim sendo, deve haver um natural ímpar p, tal que R = 2^44 * p

Substituindo R por 2^44 * p em (2), temos:

2^44 * (b/a) = 2^49 * ak + 2^44 * p

Daí,  2^44 * [ (b/a) - p ] = 2^49 * ak

Ou, [ 2^44 * (b - ap) ] / a = 2^49 * ak

Como 2^29 * ak é um inteiro postivo, a deve dividir [ 2^44 * (b - ap) ].

2^44 nao é divisível por a, uma vez que a é um natural ímpar.

Daí, conclui-se que a deve dividir (b - ap).

Para isso, é necessario e suficiente que a divida b. (CONCLUSAO 1)

Mas, de (1), a#93/a#49 = 2^44 * [ (3^93 + 4^93) / (3^49 + 4^49) ] =
= 2^44 * (b/a)

Como b é divisível por a (conclusao 1), entao [ 2^44 * (b/a)] é um inteiro
positivo, o que nos mostra que a#93 é divisível por a#49.

Assim sendo, o resto de a#93 / a#49 é ZERO.

Resposta: ZERO.

Espero ter ajudado, apesar da demora.

Saudaçoes Tricolores, Alexandre Terezan.

- Original Message -
From: "Eduardo Quintas da Silva" [EMAIL PROTECTED]
To: [EMAIL PROTECTED]
Sent: Sexta-feira, 4 de Agosto de 2000 22:21
Subject: Problema


Seja a*n = 6^n + 8^n. Calcule o resto de a*93 / a*49.

Entende-se por a*n : a índice n.




Get Your Private, Free E-mail from MSN Hotmail at http://www.hotmail.com





problema proposto

2000-08-02 Por tôpico Alexandre F. Terezan



Gostaria de propor um problema, diretamente ligado ao 
problema da dízima...

1) Seja a dízima periódica 1/97 cujo período é composto por 96 
algarismos. Pede-se:


a) Sem utilizar o método por tentativas, encontre todos os m 
possíveis para os quais 1/97 = 0,abcd..mm..vxyzabcd... , dizendo tambem, para 
cada m, quantos desses pares existem no período de 
1/97.

b) Prove que não existem 3 algarismos iguaisem sequencia 
neste período, ou seja, não há k para o qual 1/97 = 
0,abcd..kkk..vxyzabcd...

OBS: Calculando-se o período de 1/97 (o que deve levar uns 10 
minutos), a solução é óbvia. O objetivo é estimular o raciocínio, portanto, tais 
solucoes bracais sao dispensadas.

Alexandre Terezan




Re: dizima

2000-08-02 Por tôpico Alexandre F. Terezan





  Ora, para que 97H termine em 99, portanto, (7i + 7) deve 
  terminar em 9, ou seja, 7i termina em 6.
  Isto ocorre se e somente se i for um número da forma 
  10j + 6.
  
O textoacima deve ser lido como:


  Ora, para que 97H termine em 99, portanto, (7i + 7) deve 
  terminar em 9, ou seja, 7i termina em 2.
  Isto ocorre se e somente se i for um número da forma 
  10j + 6.
  
DESCULPEM O ERRO DE DIGITAÇÃO...


Re: dizima

2000-08-01 Por tôpico Alexandre F. Terezan



Caro Carlos,

Sabe-se que quandoo periodo da representacao 
decimal de 1/n possui n-1 casas decimais (como é o caso de n = 7, 17, 19, 23, 
29, 47, 59, 61, 97, etc), toda fracao r/n (com r variando de 2 a n-1) possuirá 
um período composto pelos mesmos algarismos do periodo de 1/n, na mesma ordem 
circular, mas variando a posição dos algarismos (A demonstracao se 
encontra na Eureka 1)

Trocando em miúdos, se 1/n = 
0,abcdefabcdefabcdef... há um r para o qual r/n = 
0,bcdefabcdefabcdefa... , onde o negrito representa o periodo 
de representacao decimal das fracoes.

Assim, sendo 1/97 = 
0,abcdefvxyzabcdefvxyzabcdefvxyz... , deve haver um 
k (1  k  97) tal que:

k/97 = 
0,xyzabcdefvxyzabcdefvxyzabcdefv... 

Ou seja, 1000k/97 = 100x + 10y + z + 
0,abcdef...vxyzabcdef...vxyzabcdef...vxyz...

Ou: 1000k/97 = 100x + 10y + z +1/97

E entao: (1000k - 1)/97 = 100x + 10y + z. Seja 100x + 
10y + z = H.

Assim, 1000k - 1 = 97H -- 97H = 1000(k-1) + 
999, logo os 3últimos algarismos de 97H sao 9s.

Considere H, i, j, m como inteiros nao-negativos.

Para que um 97H termine em 9, H deve ser um número da forma 
10i + 7.

Assim, 97H = 970i + 679 = 900i + 600 + 70i + 70 + 9 =100(9i + 
6) + 10(7i + 7) + 9

Ora, para que 97H termine em 99, portanto, (7i + 7) deve 
terminar em 9, ou seja, 7i termina em 6.
Isto ocorre se e somente se i for um número da forma 10j 
+ 6.

97H = 970i + 679 = 9700j + 5820 + 679 = 1000(9j + 6) + 100(7j 
+ 4) + 99

Para que 97H termine em 999, portanto, (7j + 4) deve terminar 
em 9, ou seja, 7j termina em 5.
Isto ocorre se e somente sej for um número da forma 10m 
+ 5.

Assim, H = 10i + 7 = 10(10j + 6) + 7 = 100j + 67= 
100(10m + 5) + 67 = 1000m + 567. Para todo m inteiro nao-negativo, portanto, 97H 
terminará em 999, o que nos dá um k inteiro.

Para m  0, no entanto, H  1566 e, portanto, 97H  
151902. 
Assim, 1000k  151903 -- k  151 , o que 
nao é possivel, pois0  k  98 

Logo, m = 0 -- H = 567 -- 1000k= 55000 
-- k = 55

Como H =100x + 10y + z , com x,y,z 
inteiros nao-negativos menores que 10:

100x + 10y + z = 567 -- x = 5 y = 
6 z = 7

Portanto, o último algarismo do período de 1/97 
é 7
o 
penúltimo algarismo do período de 1/97 é6
o 
antepenúltimo algarismo do período de 1/97 é5

Abraços,Terezan

Espero ter 
ajudado. 



- Original Message - 

  From: 
  Carlos Roberto de Moraes 
  To: [EMAIL PROTECTED] 
  Sent: Terça-feira, 1 de Agosto de 2000 
  18:29
  Subject: dizima
  
  Alguem pode me ajudar? Ao escrevermos 1/97 na 
  forma decimal, obtemos uma dízima periódica com 96 algarismos no período. Como 
  determinar os três últimos algarismos desse 
período?


Re: dizima

2000-08-01 Por tôpico Alexandre F. Terezan



Só para deixar bem claro, k  1, pois k = 1 seria a própria 
fração 1/97 e para k = 0, k/97 = 0.

k  97, pois para k = 97, k/97 = 1 e para k  97, k/97 
 1, nao sendo da forma 0,abcdef...

  - Original Message - 
  From: 
  Carlos Roberto de Moraes 
  To: [EMAIL PROTECTED] 
  Sent: Terça-feira, 1 de Agosto de 2000 
  18:29
  Subject: dizima
  
  Alguem pode me ajudar? Ao escrevermos 1/97 na 
  forma decimal, obtemos uma dízima periódica com 96 algarismos no período. Como 
  determinar os três últimos algarismos desse 
período?


Re: Questão das Olimpíadas

2000-07-28 Por tôpico Alexandre F. Terezan




Caro mtu,

 Sua resolução me parece perfeitamente correta, mas 
deve-se lembrar que a questão poderia ser discursiva e o valor mínimo de q 
poderia ser alto o suficiente para que tal resolução por "tentativa" 
nãofosse eficiente e muito menos prática.

Uma maneira simples de resolver a questão seria:

Seja p =q - k (k natural maior q zero), já que p 
 q(pois p/q  11/15  1)

Então 7/10  (q - k)/q  11/15

Daí, 7q  10q - 10k -- 3q  10k -- q  
10k/3
 15q - 15k  11q -- 4q 
 15k -- q  15k/4

Assim sendo, 10k/3 q  15k/4
Ou, 3k + k/3  q  3k + 3k/4 (CONDIÇÃO 1)

Como 3k e q são ambos naturais, conclui-se que:a + [k/3] = [3k/4], 
sendo a natural maior do que zero.

Ora, como se procura o menor q (logo, o menor k natural satisfazendo a 
condição 1), entao a = 1 e assim:

[3k/4]= 1 -- 3k/4  1 -- k  4/3
[k/3] = 0 --k/3  1 -- k  3

4/3  k  3 -- k = 2.

Assim, da condição 1, vem: 6 + 2/3  q  6 + 6/4 -- 6 
+ 2/3  q  7 + 1/2 --q = 7

Para todo k q satisfaça a condição 1, encontramos p/q tal que 7/10  p/q 
 11/15

k =3 -- q = 11 -- p = 8 -- 7/10  8/11  
11/15
k = 4 -- q = 14-- p =10 -- 7/10  10/14  
11/15
k = 5 -- q = 17 -- p = 12 -- 7/10  12/17  11/15
k = 5 -- q = 18 -- p =13 -- 7/10  13/18  
11/15
etc etc etc

Abraços, Alexandre Terezan

- Original Message - 
From: "mtu" [EMAIL PROTECTED]
To: [EMAIL PROTECTED]
Sent: Sexta-feira, 28 de Julho de 2000 03:51
Subject: Re: Questão das Olimpíadas
José Alvino wrote: Olá pessoal!Sou novato aqui 
na lista e gostaria que alguém me ajudasse numa questão ou me informasse 
onde posso encontrar sua resolução. É a questão 10 da 1a fase junior de 
97: Se p e q são inteiros positivos tais que 7/10  p/q  11/15 , 
o menor valor que q pode ter é: A) 6 B) 7 C) 25 D) 30 E) 
60 Agradeço antecipadamente.Acho q a resposta seria B, p/q 
deve ser um numero entre 0,7 e 0,733...,entao pensei da seguinte forma se 
usar q = 6, joguei como possibilidadepara p os numeros 5 e 4, 5/6 eh entao 
um numero maior que 0,733.. e 4/6eh um numero menor q 0,7, ou seja, q != 6, 
repeti o processo com 7 edescobri q se p=5 e q=7(5/7) sera um numero q se 
encaixa nadesigualdade, ou seja, dos numeros apresentados o menor que 'q' 
podeassumir eh 7.Nao sei se esse eh o meio mais pratico de resolver o 
problema, mas meparece 
correto... 
[]'s


dúvida

2000-07-28 Por tôpico Alexandre F. Terezan



Qual a demonstração das desigualdades das médias aritmética, 
geométrica e harmônica?


Re: teoria dos inteiros

2000-07-26 Por tôpico Alexandre F. Terezan



ERRATA: Uma pequena correção, que pouco influi e talvez tenha 
passado despercebida.Onde está:

Então 100c + 10b + a - 100a - 
10b - a = (k-1) * ABC
Ou, 99 (c - a) = (k-1) * ABC

Obviamente, o correto é: 
Então 100c + 10b + a - 100a - 10b -c = 
(k-1) * ABC
Ou, 99 (c - a) = (k-1) * 
ABC

  - Original Message - 
  From: 
  Alexandre F. Terezan 
  To: [EMAIL PROTECTED] 
  Sent: Terça-feira, 25 de Julho de 2000 
  21:58
  Subject: Re: teoria dos inteiros
  
  Essa é a primeira vez q escrevo para a lista, portanto, 
  saudações a todos.
  
  SejaXYZ a representação de 100x + 10y + 
z.
  E seja = a notação para menor ou igual a
  
  Assim, se CBA é múltiplo de ABC, então existe um inteiro 
  positivo k tal que:
  
  CBA = ABC * k , para0  k  10 
  , ou CBA - ABC = (k-1) * ABC
  
  Então 100c + 10b + a - 100a - 10b - a = (k-1) * 
  ABC
  Ou, 99 (c - a) = (k-1) * ABC
  
  Assim, temos 2 casos a estudar:
   i) c = a -- k = 1
   ii) c  a -- k  1 
  (obviamente se c fosse menor do q a, então ABC  CBA, 
  impossível.
  
  Caso i:
  
   Neste caso, para todo c = a, k = 1 
  satisfaz o enunciado, ou seja, CBA = ABC * 1 = ABC. Como 0  c = 
  9 e 0 = b = 9, são90 as situações 
  possíveis.
  
  Casoii:
  
   Neste caso, (k-1) * ABC = 3^2 * 11 * (c - 
  a). Note que (k - 1)  9, pois k  10. Assim, ABC é 
  obrigatoriamente múltiplo de 33 (um fator 3 e um fator 11).
   Note também que a  5 , uma vez que do 
  contrário CBA = k * ABC possuiria mais de 3 algarismos, para k  1, 
  impossível.
   Além disso, se CBA for ímpar (a ímpar), 
  ABC é necessariamente ímpar (c ímpar). Por outro lado, se ABC é par (c par), 
  então CBA também é par (a par).
   Dos critérios de divisibilidade por 3 e 
  por 11, tira-se que:
   1) a + b + c = 3r (r inteiro 
  positivo menor do que 8)
   2) - a + b - c = 11s (s= -1 
  ou s = 0 -- esta restrição se deve ao fato de serem a,b,c inteiros tais 
  que 0  a  5, 0 = b  10e 0  c  
  10)
  
   Somando-se as 
  equações 1 e 2, obtemos: 2b = 3r + 11s 
   Para s = -1, vem: 2b = 3r - 
  11 e a + c = b + 11.
  Lembrando que c  a, apenasum terno 
  (a,b,c)seria uma possívelsolução, o terno(4,2,9). Como 924 
  não é múltiplo de 429, essa solução é falsa. 
  
   Para s = 0, vem: 2b = 3r 
  e a + c = 3r/2. 
  4 ternos (a,b,c) seriam possíveis soluções: (2,6,4) , 
  (1,6,5) , (4,9,5) e(2,9,7). No entanto, nenhuma dessas possíveis 
  soluções é verdadeira, ou seja, CBA não é múltiplo de ABC para nenhum 
  desses ternos.
  
  
  Ora, então todas as soluções se resumem ao caso i, 
  totalizando 90 soluções , para todo 0  c = a  10 (9 
  casos) e 0 = b  10 (10 casos) -- 9 * 10 = 90 
soluções.
  
  Abraços, Alexandre Terezan
  
  - Original Message --
  
From: 
Filho 

To: discussão de 
problemas 
Sent: Segunda-feira, 24 de Julho de 
2000 08:38
Subject: teoria dos inteiros

Quantos são os inteiros positivos de três 
algarismos abc, com a e c ambos diferentes de zero, tais que cba é múltiplo 
de abc ?